You are on page 1of 139

NEET PG

March - 2023 Solved Paper

Important Note –

To get daily updates on High Yield Topics / Concept Cards / Quizzes / Masterclasses / Clinical Images & Question Join
our FREE Whats App group

To Join goto www.medicoapps.org/groups

Or Scan the QR Code Below


Anatomy
Q. 1 - A patient has been presented with progressive pain in a swelling below the ear lobule. Involvement of
which of the following structures is responsible for the pain?

A. Facial nerve B. Auriculotemporal nerve

C. Great auricular nerve D. Deep cervical fascia

Correct Answer - D
Explanation
Patient has an infection or inflammation of the submandibular gland, which is located below the ear lobule and
is surrounded by the deep cervical fascia. This condition is called submandibular sialadenitis and can cause
progressive pain and swelling.

Other possible causes of pain and swelling in this area include an abscess, lymphadenitis (infection of the lymph
nodes), or a tumor.

Q. 2 - A 5 year old child with features of acute tonsillitis is complaining of ear pain. The nerve responsible for
ear pain is.

A.Greater auricular nerve B. Tympanic branch of glossopharyngeal nerve


C. Auricular branch of vagus D. Mandibular nerve

Correct Answer - B
Explanation
The glossopharyngeal nerve (CN IX) is responsible for providing sensory innervation to the tonsils, posterior
third of the tongue, and the oropharynx.

The tympanic branch of the glossopharyngeal nerve supplies sensory innervation to the middle ear and is
responsible for transmitting pain signals from the middle ear to the brain.

In cases of acute tonsillitis, the infection may spread to the middle ear via the Eustachian tube, causing
inflammation and pain in the middle ear. This is known as acute otitis media and can be a complication of
tonsillitis in some cases.

Therefore, it is possible that the 5-year-old child with acute tonsillitis is experiencing referred pain to the ear
due to involvement of the tympanic branch of the glossopharyngeal nerve.

2
Q. 3 - Congenital anomaly shown in the following diagram relates to which of the following pharyngeal arch
artery.

A. Falciform ligament B. Median umbilical ligament


C. Medial umbilical ligament D. Lateral umbilical ligament

Correct Answer - B
Explanation
The median umbilical ligament is a fibrous cord that runs vertically in the midline of the abdominal wall,
extending from the umbilicus to the urinary bladder. This ligament is the remnant of the embryonic structure
called the urachus, which connects the urinary bladder to the umbilical cord during fetal development.

Congenital anomalies related to the median umbilical ligament are rare but can occur. One such anomaly is a
persistent urachus, which happens when the urachus fails to close properly after birth, leading to a connection
between the bladder and the umbilicus.

This can cause urine leakage from the umbilicus and increase the risk of urinary tract infections.Another
congenital anomaly related to the median umbilical ligament is an urachal cyst. This occurs when the urachus

3
fails to close and results in the formation of a fluid-filled sac or cyst near the umbilicus. Urachal cysts are usually
asymptomatic but can become infected, leading to pain and discomfort.

In rare cases, the median umbilical ligament can also be associated with other congenital anomalies such as
bladder exstrophy, a condition in which the bladder protrudes through the abdominal wall.

Treatment for these congenital anomalies related to the median umbilical ligament typically involves surgical
removal of the affected tissue or structure

Q. 4 - In a patient of liver cirrhosis anastomotic blood flow increase in

A. Left gastric vein & esophageal veins B. Middle colic vein & left colic vein
C. Sigmoidal & superior rectal vein D.Superior & inferior phrenic vein

Correct Answer - A
Explanation
An increase in anastomotic blood flow in the left gastric vein and esophageal veins in a patient with liver
cirrhosis may be a sign of portal hypertension.

Portal hypertension is a common complication of liver cirrhosis, where there is increased resistance to blood
flow through the portal vein that carries blood from the gastrointestinal tract, spleen, and pancreas to the liver.

As a result of the increased resistance, blood flow may be diverted through collateral vessels, such as the left
gastric vein and esophageal veins, which can enlarge and become varicose.

These varices can be prone to bleeding and can be a serious complication of portal hypertension.

4
Physiology

Q. 5 - One transport system across cell membrane is given below. The process indicates which of the
following?

A. Simple diffusion B. Facilitated diffusion


C. Secondary active transport D. Primary active transport

Correct Answer - B
Explanation
Facilitated diffusion is the passive movement of molecules along the concentration gradient.

It is a selective process, i.e., the membrane allows only selective molecules and ions to pass through it.
It, however, prevents other molecules from passing through the membrane.

Q. 6 - The clearance of substances X, Y, Z are studied at different concentrations in the blood (diagram below).
Which of the following is correctly matched?

A. X- glucose, Y- HCO3-, Z- Inulin B. X- Inulin, Y- HCO3-, Z- Inulin


C. X- glucose, Y- HCO3-, Z- PAH D. X- Inulin, Y- PAH, Z- glucose

5
Correct Answer - D

Explanation
Increasing the plasma concentration of inulin the clearance of inulin (in the urine) does not change.
It remains a flat, straight line.

For glucose at low plasma levels of glucose, ALL glucose is reabsorbed. aka none enters the urine (unless
diabetic) Hence the urinary clearance of glucose in zero at lower concentration of glucose. However as the
concentration of glucose increases beyond the Transport Maximum value, then urinary clearance of glucose
increase with serum level of glucose and then becomes stationary. However the clearance of glucose will always
be below that of inulin because some amount of glucose will always be reabsorbed.

Creatinine has a curve similar to inulin however, creatinine curve will always be higher than inulin. This is
because trace amounts of creatinine are always being secreted from the peritubular capillaries into the tubular
fluid and then excreted out via the urine. Unlike inulin which is neither reabsorbed or secreted.

PAH - At low plasma levels the clearance of PAH equals that of renal plasma flow. However As the plasma (PAH)
concentration increases the clearance of PAH decreases

Q. 7 - In 35 year old patient due to intravascular homolysis, serum K+ increased to 5.5 mEq/L. What will
happen to resting membrane potential in this patient

A. More positive B. More negative


C. Remain unchanged D. Zero

6
Correct Answer - A
Explanation
First, in the setting of hyperkalemia, the resting membrane potential is shifted to a less negative value, that is,
from −90 mV to −80 mV, which in turn moves the resting membrane potential closer to the normal threshold
potential of −75 mV, resulting in increased myocyte excitability.

Increased extracellular potassium levels result in depolarization of the membrane potentials of cells due to the
increase in the equilibrium potential of potassium.

This depolarization opens some voltage-gated sodium channels, but also increases the inactivation at the same
time.

Since depolarization due to concentration change is slow, it never generates an action potential by itself;
instead, it results in accommodation.

Above a certain level of potassium the depolarization inactivates sodium channels, opens potassium channels,
thus the cells become refractory.

This leads to the impairment of neuromuscular, cardiac, and gastrointestinal organ systems.

Of most concern is the impairment of cardiac conduction, which can cause ventricular fibrillation and/or
abnormally slow heart rhythms

Q. 8 - If dorsal column is hit from lateral side which of the following fibers would be affected first-

A. Cervical B. Sacral
C. Thoracic D. Lumbar

Correct Answer - A
Explanation
The dorsal column carries sensory information from various parts of the body, including touch, vibration, and
proprioception. The fibers of the dorsal column that originate from the cervical region are located more
laterally within the column. Therefore, if there is an injury or damage to the dorsal column from the lateral side,
the cervical fibers would be the first to be affected.
7
Q. 9 - Action potential is recorded from a mammalian neuron, and marked with different letter. Which of
following pair is correct-

A. Point A- threshold B. Point B to C- Na+ channel opening


C. Point D- K+ channel closure D. Point E- Absolute refractory period

8
Correct Answer - B

Q. 10 - Spirometer recording was given in the following diagram. The part marked with the arrow, indicates-

A. Residual volume B. FRC


C. ERV D. Closing volume

Correct Answer - B

9
Q. 11 - In cases of infant respiratory distress syndrome, there is deficiency of which lipid

A. Lecithin B. Sphingomyelin
C. Phosphatidylserine D. Phosphatidylethanolamine

Correct Answer - A
Explanation
Infants with idiopathic respiratory distress syndrome (RDS) almost exclusively are prematurely born, and their
lungs lack adequate surface activity and are deficient in the principal surface-active component, lecithin.

Q. 12 - If surfactant is deficient in lungs, what happens-

A. Surface tension decreases, compliances increases


B. Surface tension increases, compliance decreases
C. Both surface tension & compliance increases
D. Both surface tension & compliance decreases

Correct Answer - B
Explanation
It decreases surface tension.
It permits lungs to be more easily inflated and increases compliance

Q. 13 - The phases of ventricular muscle action potential is given below. Which of the following is correctly
matched

10
A. Calcium channel open in phase 2 B. Potassium channel closes in phase 0
C. Transient sodium channel open in phase 3 D. Calcium channel closes in phase 4

Correct Answer - A

Q. 14 - Vitamin K dependent clotting factor


A. Factor I B. Factor II
C. Factor III Factor IV

Correct Answer - B
Explanation
The clotting factors involved are FII, FVII, and FIX.

11
Biochemistry

Q. 15 - A family consuming predominantly of polished rice would present with which of the following vitamin
deficiencies. Match the vitamin deficiency with the investigation used to detect the deficiency?

A. Riboflavin with Glutathione reductase B. Pyridoxine with Xanthurenic acid


C. Riboflavin with Transketolase D. Thiamine with Transketolase

Correct Answer -D
Explanation
A deficiency of thiamine in the diet Young adults with severe anorexia and people whose diet consists mainly of
highly processed carbohydrates (such as polished white rice, white flour, and white sugar) may not consume
enough thiamin. Polishing rice removes almost all of the vitamins.

Beri-Beri is caused by thiamine (Vitamin B1) deficiency. It's usually found in places where polished rice is a
staple of the diet.

Transketolase is a thiamine pyrophosphate (vitamin B1)-dependent enzyme, and, along with pyruvate
dehydrogenase and α-ketoglutarate dehydrogenase of the tricarboxylic acid cycle, the enzyme is affected by
thiamine deficiency (beriberi).

Q. 16 - When there is low insulin: glucose ratio, which enzyme is stimulated?

A. Lipoprotein lipase B. Hormone sensitive lipase


C. Phosphofructokinase I D. Glycogen synthase

Correct Answer - B
Explanation
Insulin inhibits the action of hormone-sensitive lipase and prevents hydrolysis of the triglycerides in adipose
tissue and release of fatty acids into the circulating blood.

Q. 17 - A 4 year old child easy fatiguability. The mother also complained that the child was more hungry
between meals, and the child recovers after food. Liver examination revealed no glycogen
12
A. Branching enzyme B. Debranching enzyme
C. Glucose 6 phosphatase D. Glycogen synthase

Correct Answer - D
Explanation
Glycogen storage diseases (GSDs) are inherited inborn errors of carbohydrate metabolism. Disorders of
carbohydrate metabolism that result in abnormal storage of glycogen.

The absence or malfunction of liver glycogen synthase due to mutations in the GYS2 gene will prevent glycogen
from being synthesized in the liver & this is the cause of GSD.

Common presenting symptoms include growth retardation/poor weight gain in children, exercise intolerance,
hypoglycemia, hepatomegaly, low muscle tone, acidosis, and hyperlipidemia

13
Microbiology

Q. 18 - A 45 year old male presents to OPD with a chief complaint of abdominal pain associated with diarrhea
for last 3 days. He also reports fatigue, anorexia, weight loss. O/E reveals a BP of 130/70 mm Hg & a regular
pulse rate of 70 beats/ min along with jaundice. The imaging of the liver with USG shows intra & extra
hepatic dilatation & strictures with intraductal pigmented stones. What is the causative agent?

A. Enterobius (pinworm) B. Stronglyoides stercoralis


C. Ancylostoma duodenale D. Clonorchis sinensis

Correct Answer - D
Explanation
Patients with a very heavy parasitic load (>20 000) can present with symptoms compatible with acute
cholangitis, including jaundice, right upper quadrant abdominal pain, nausea, vomiting, anorexia, malaise, and
fevers.

C. sinensis induces hyperplastic changes of the gallbladder wall resulting in decreased gallbladder contractions
and the subsequent precipitation of bilirubinate and mucin on the parasitic eggs.

pancreatitis and liver abscesses appear more frequently in patients with clonorchiasis. Chronic obstruction and
strictures of the bile ducts can also result in bile stasis and recurrent ascending cholangitis mediated by
Escherichia coli.

On ultrasound, diffuse dilatation of intrahepatic bile ducts is commonly seen in patients infected with
Clonorchis sinensis.

Q. 19 - There is a sewer worker presented with high grade fever, jaundice with conjunctival suffusion. What is
the next investigation to diagnose it?

A. Microscopic agglutination test B. Paul Bunnell


C. Widal test D. Weil Felix test

14
Correct Answer - A
Explanation
Leptospirosis
Leptospirosis, also called Weil disease, is the most common zoonotic infection in the world. Leptospirosis is
caused by an infection with the spirochete bacterium Leptospira.

It is most often spread through exposure to the urine of infected animals either from direct contact or from
contact with soil or water contaminated by the urine.

The diagnosis of leptospirosis is made by growth in a specialized culture or microscopic agglutination test.

Q. 19 - Boys hostel get infected as hostelers develop polymorphous rashes with fever?

A. Isolation for 6 days


B. Isolation for 6 days and give acyclovir
C. Isolation for 14 days & give IV immunoglobulin within 48 hours
D. Isolation for 6 days & give IV immunoglobulin within 72 hours

Correct Answer - A
Explanation

Measles, also known as rubeola, is a preventable, highly contagious, acute febrile viral illness
The initial inflammation leads to symptoms of coryza, conjunctivitis, and cough. The appearance of fever
coincides with the development of viremia. The skin rash occurs after dissemination and is due to perivascular
and lymphocytic infiltrates.

The initial humoral response consists of IgM antibody production, which is detectable 3 to 4 days after the rash
appears and can persist for 6 to 8 week
Patients with measles should remain in Airborne Precautions for 4 days after the onset of rash (with onset of
rash considered to be Day 0).

15
Q. 20 - Rotavirus diagnosis?

A. Antigen in stool B. Ab in stool


C. Antibody in serum D. Light microscopy of stool sample

Correct Answer - A
Explanation
Polyacrylamide gel electrophoresis detects rotavirus RNA extracted directly from stool specimens

Q. 21 - What is true for trichomonas?

A. Twitching motility is seen on wet mount B. Cannot be grown in culture


C. Darting motility is seen D. Cysts are seen on wet mount

Correct Answer - A
Explanation
Women with trichomoniasis will often present with vaginal discharge, painful intercourse, urinary tract infection
symptoms, vaginal itching, or pelvic pain.

It is a motile organism that lives in the lower genitourinary tract of females and the prostate and urethra of
men.

The most common diagnostic testing performed is wet prep microscopy.

Trichomonads are motile organisms with flagella and can be seen moving in the preparation when viewed with
a microscope.

Wet mount
A physiological saline preparation of vaginal secretions
Trichomonads can be identified by size (similar to WBC), shape and quivering/twitching motility
Require ~104 trichomonads/ml vaginal fluid

16
Q. 22 - A child with fever, headache and neck stiffness. On CSF examination, it reveals low glucose & gram
negative pleomorphic organisms. What is the characteristic of
organism on culture?

A. Alpha hemolysis of sheep RBC B. Satellitism in presence of Staphylococcus aureus


C. Non lactose fermentation on Mc Conkey agar D. Bacitracin sensitive

Correct Answer - B
Explanation
Acute bacterial meningitis is the most common infection of CNS
Recently, more than 80% of the cases were due to the triad - Streptococcus pneumoniae, Haemophilus
influenzae type b, and Neisseria meningitidis, which were transmitted through respiratory secretions.

Q. 23 - A patient with fever & swelling is positive for Filarial Antigen. What is the next step?

A. DEC provocation test B. LN biopsy


C. BM aspiration D. Peripheral blood smear to identify microfilaria

17
Correct Answer - A
Explanation
DEC (DIETHYLCARBAMAZINE) PROVOCATION TEST

DEC (diethylcarbamazine) provocation test is used in the diagnosis of lymphatic filariasis

The usual method for confirming filariasis is by demonstrating microfilariae in night blood specimens ( due to
the nocturnal periodicity)

Q. 24 - A patient presented HSM. O/E pallor was present. Rapid antigen test was positive for HRP- 2 Ag.
Identify the species of plasmodium causing disease?

A. Falciparum B. Malariae
C. Ovale D. Vivax

Correct Answer - A
Explanation
Severe and fatal malaria is predominantly caused by Plasmodium falciparum.

Symptoms are generally non-specific, including fever, chills, myalgia, headache, anorexia, pallor and cough
Rapid diagnostic tests are commonly used in addition to blood slides and are useful alternatives in settings
where a microscopic diagnosis is non-reliable or infeasible.

They are immune-chromatographic tests that most commonly identify either malaria antigens (e.g., P.
falciparum histidine-rich-protein 2 (PfHRP2)) or enzyme called Plasmodium lactate dehydrogenase (pLDH).

Q. 25 - A man from Bihar presents with fever & skin lesions. On microscopy an organism with kinetoplast
was seen. What's the causative agent?

A. Leishmania tropica B. Toxoplasma gondii


C. Babesia microti D. Malaria

18
Correct Answer - A
Explanation
Leishmania organisms are dimorphic protozoa existing in two distinct morphologies within sand fly vectors and
mammalian hosts, respectively.

Within 24 to 48 hours after infection, intracellular promastigotes transform into oval or round amastigotes
(1.5–3 µm × 3–5 µm) that lack a visible flagellum on light microscopy. Amastigotes have a distinct, rod-shaped
structure called a kinetoplast, which is a specialized mitochondrial structure. Visualization of a kinetoplast, as
seen, confirms the diagnosis of leishmaniasis.

Q. 26 - A patient with multiple sexual partners manifested with fever & jaundice. SGPT was high. HBsAg &
HBeAg were +ve. What is the treatment?

A. Lamivudine B. Peg interferon


C. Combination of peg interferon + lamivudine D. Tenofovir >54 weeks

Correct Answer - C
Explanation
Hepatitis B
It is transmitted parenterally and sexually when individuals come in contact with mucous membranes or body
fluids of infected individuals.

Symptoms of anorexia, malaise, and fatigue which are the most common initial clinical symptoms.
Once these patients progress to the icteric phase, they develop jaundice and painful hepatomegaly.

In patients who have an acute hepatitis B virus infection, the first serum marker to appear is the hepatitis B
surface antigen (HBsAg).

Hepatitis B e antigen (HBeAg) is also present and indicates that the virus is replicating.

First-line monotherapy agents include pegylated interferon alfa-2a (PEG-IFN) and oral nucleoside or nucleotide
analogs, including tenofovir or entecavir, lamivudine

19
Q. 27 - Owl eye inclusion bodies?

A. CMV B. Toxoplasma
C. HSV D. EBV

Correct Answer - A
Explanation
Owl's eye appearance of inclusion bodies, which is highly specific for cytomegalovirus infection.

Q. 28 - A 50 year old man in Delhi complaints of fever, headache, joint pain, petechial rash and bleeding
gums. Blood profile Hb 10, WBC- 3000, Platelet 20000 Diagnosis?

A. Dengue B. Scrub typhus


C. Babesiosis D.typhoid

Correct Answer - A
Explanation
Dengue is a mosquito-transmitted virus and the leading cause of arthropod-borne viral disease in the world. It
is also known as breakbone fever due to the severity of muscle spasms and joint pain, dandy fever, or seven-day
fever because of the usual duration of symptoms.

It is described as a fever that remits at least for one day, and the next fever spike starts, which lasts at least for
one more day. Associated symptoms include facial flushing, skin erythema, myalgias, arthralgias, headache, sore
throat, conjunctival injection, anorexia, nausea, and vomiting. For skin erythema, a general blanchable macular
rash occurs in the first one to two days of fever and the last day of fever. Or, within 24 hours, a secondary
maculopapular rash can develop.

Common laboratory findings include thrombocytopenia, leukopenia, elevated aspartate aminotransferase.

Q. 29 - A patient with painless genital ulcer & non tender inguinal lymphadenopathy?

A. Syphilis B. Chancroid
C. Granuloma inguinale D. Genital TB

20
Correct Answer - A
Explanation
Syphilis is a systemic bacterial infection caused by the spirochete Treponema pallidum
The classic primary syphilis presentation is a solitary non-tender genital chancre in response to invasion by the
T. pallidum.

The clinical manifestations of secondary syphilis result from hematogenous dissemination of the infection and
are protean: condyloma lata (papulosquamous eruption), hands and feet lesions, macular rash, diffuse
lymphadenopathy, headache, myalgia, arthralgia, pharyngitis, hepatosplenomegaly, alopecia, and malaise
Primary syphilis appears 10 to 90 days after exposure to the infection and comprises a painless, indurated ulcer
(chancre) at the site of inoculation with the T. pallidum. HIV patients usually develop multiple chancres. These
lesions resolve without treatment in 3-6 weeks. Regional lymphadenopathy is common and consists of rubbery
lymph nodes.

Q. 30 - A patient with multiple sexual partners manifested with urethral discharge. Gram stain showed the
following findings. Treatment is?

A. Ceftriaxone B. Ampicillin
C. Azithromycin D. Doxycycline

Correct Answer - A
Explanation
Neisseria gonorrhoeae
Neisseria gonorrhoeae, an obligate human pathogen, is a sexually transmitted disease
N. gonorrhoeae STI treatment for urogenital infections in males and females most commonly consists of dual
therapy with a single intramuscular or intravenous dose of 500 mg of ceftriaxone in conjunction with
doxycycline 100 mg orally twice a day for 7 days.In patients 150 kg or more, 1 g of ceftriaxone should be given.

21
Q. 31 - A HIV patient with pneumonia negative for BAL KOH wet mount but Gomori Methenamine stain
showed crescent shape black color cysts and the organism was non- cultivable.
Diagnosis?

A. Cryptococcus B. Pneumocystis
C. Coccidioides D. Paracoccidioides

Correct Answer - B
Explanation
Pneumocystis carinii pneumonia (PCP), now referred to as Pneumocystis jirovecii pneumonia, is a fungal
infection that most commonly affects the immunocompromised and, in some cases, can be severely
life-threatening.

Stains like Giemsa, crystal violet, and Diff-Quick can be used to detect both the cyst and trophozoite. In
addition, the walls can be stained with Toluidine blue or methenamine silver.

The cysts often collapse, forming crescent-shaped bodies visible in stained tissue.

22
Pharmacology

Q. 32 - The patient with increased uric acid is taking medication for gout. Which of the following drug act by
inhibiting uric acid synthesis?

A. Probenecid B. Allopurinol
C. Colchisine D. Diclofenac

Correct Answer - B
Explanation
The class of drugs that acts by inhibiting uric acid synthesis is called xanthine oxidase inhibitors. These drugs
work by blocking the enzyme xanthine oxidase, which is responsible for the conversion of hypoxanthine to
xanthine and then to uric acid.

The most commonly used xanthine oxidase inhibitor for the treatment of gout is allopurinol. Allopurinol is a
medication that is taken orally and works by reducing the production of uric acid in the body. By inhibiting
xanthine oxidase, allopurinol reduces the amount of uric acid that is produced, which can help to prevent gout
attacks.

Another xanthine oxidase inhibitor that is sometimes used for the treatment of gout is febuxostat. Febuxostat
works in a similar way to allopurinol, but it is a newer medication that may be better tolerated by some
patients.

Both allopurinol and febuxostat are effective in reducing the production of uric acid and can help to prevent
gout attacks. However, they are not effective at treating acute gout attacks and are typically used for long-term
management of gout to prevent future attacks.

Q. 33 - A person was given digoxin developed toxicity & levels were 4ng/ml. How much time will it take to
reach the therapeutic concentration of 1ng/ml, Digoxin has a half life of 40 hours.

A. 80 hours B. 40 hours
C. 160 hours D. 120 hours

23
Ans. 4ng/ml (1T1/2)→ 2ng/ml (2T1/2) → 4ng/ml
T ½ = 40 hours
2 T ½ = 40 x 2 hours = 80 hours

Q. 34 - A person was on cancer chemotherapy & developed thrombocytopenia. Which of the following drug is
used to recover from thrombocytopenia?

A. Epoetin B. Oprelvekin
C. Filgrastim D. Amifostine

Correct Answer - B
Explanation
Oprelvekin (also known as interleukin-11) is a medication that can be used to treat thrombocytopenia caused
by chemotherapy in cancer patients. It works by stimulating the production of platelets in the bone marrow,
which can increase the platelet count in the blood. Oprelvekin is administered by injection under the skin and is
typically given daily for up to 21 days after chemotherapy. It's important to note that like any medication,
Oprelvekin can cause side effects, such as fluid retention, nausea, vomiting, and allergic reactions. Therefore,
it's important to discuss the potential risks and benefits of this medication with a qualified healthcare provider
before starting treatment.

Q. 35 - Which of the following is ACE inhibitors and also neprilysin inhibitor

A. Omapatrilat B. Sacubitril
C. Enalapril D. Candesartan

Correct Answer - A
Explanation
Omapatrilat is a medication that was developed as a dual inhibitor of both ACE (angiotensin-converting
enzyme) and neprilysin. It was initially approved for the treatment of hypertension (high blood pressure), but its
approval was withdrawn due to safety concerns.

Although omapatrilat was effective in reducing blood pressure, studies showed that it increased the risk of
angioedema (severe swelling of the face, lips, tongue, or throat), which can be life-threatening. In addition,
omapatrilat was associated with an increased risk of renal dysfunction (kidney problems) and hyperkalemia
(high potassium levels).

24
Q. 36 - DOC for Dialysis Disequilibrium syndrome ?

A. Ethacrynic acid B. Mannitol


C. Aspirin D. Bumetanide

Correct Answer - B
Explanation
The drug of choice (DOC) for dialysis disequilibrium syndrome (DDS) is prevention through slow and cautious
dialysis.

Teatment involves several measures, such as stopping or slowing down the dialysis procedure, reducing the
patient's blood pressure, administering oxygen, and managing other symptoms such as seizures and headache.

In some cases, medications such as mannitol or hypertonic saline may be used to reduce cerebral edema and
intracranial pressure, which can be elevated in DDS.

Q. 36 - A patient is planned for cardiac surgery. What is the most appropriate preoperative antibiotic?

A. Doxycycline B. Cephazolin
C. Penicillin G D. Erythromycin

Correct Answer - B
Explanation
Cephazolin is a commonly used antibiotic for prophylaxis in patients undergoing cardiac surgery, and it is
considered appropriate for preoperative antibiotic prophylaxis.

The goal of preoperative antibiotic prophylaxis is to reduce the risk of surgical site infections (SSIs) by
preventing the growth of bacteria that may be present on the patient's skin or in the surrounding environment.
Cephazolin is a first-generation cephalosporin antibiotic that is effective against a broad range of gram-positive
bacteria, including Staphylococcus aureus, which is a common cause of SSIs.

The timing and duration of antibiotic prophylaxis may vary depending on the type of surgery, but in general, a
single dose of cephazolin is given intravenously within 60 minutes before the surgical incision. In some cases,
additional doses may be given during prolonged surgeries or in patients with a high risk of infection.

25
It's important to note that the use of antibiotics for prophylaxis should be based on local guidelines and
protocols, and should only be used when appropriate. Overuse or inappropriate use of antibiotics can
contribute to the development of antibiotic resistance, which is a growing public health concern.

Q. 37 - A child was confused & looked irritable. He had not passed urine since morning, photophobia & had
eaten something while playing in field. Which is the likely poisoning & antidote.

A. Datura-pyridostigmine B. Datura-physostigmine
C. Yellow oleander- pralidoxime D. Yellow oleander- physostigmine

Correct Answer - B
Explanation
The symptoms described, including confusion, irritability, lack of urine output, photophobia, and ingestion of a
plant while playing in a field, suggest that the child may have been poisoned with Datura, a plant that contains
various toxic alkaloids.

Physostigmine is an antidote that can be used to treat poisoning with Datura. It works by inhibiting the activity
of acetylcholinesterase, an enzyme that breaks down acetylcholine, a neurotransmitter that is affected by the
toxic alkaloids in Datura.

Therefore, datura-physostigmine is a likely scenario of poisoning in this case, and prompt medical attention is
necessary.

Treatment may include the administration of activated charcoal to reduce absorption of the toxins, supportive
care to manage symptoms such as agitation and seizures, and the use of physostigmine as an antidote.

It's important to seek medical attention immediately if poisoning is suspected.

Q. 38 - A patient with OP poisoning was started on atropine infusion & pralidoxime. He develops high grade
fever after 2 hours. What is the likely cause of fever?

A. Side effects of oxime B. Side effects of atropine


C. Due to OP poisoning D. Due to increased secretions

26
Correct Answer - B
Explanation
It is possible that the high-grade fever is a side effect of the atropine. Atropine is an anticholinergic drug, which
means it blocks the action of acetylcholine, a neurotransmitter that is involved in many bodily functions,
including regulating body temperature.

When atropine blocks acetylcholine in the body, it can disrupt the body's normal thermoregulatory
mechanisms, leading to an increase in body temperature. This can result in fever, especially at high doses or if
the patient is sensitive to the drug.

Q. 39 - What is the treatment of Aspirin overdose?

A. Naloxone B. N acetylcysteine
C. Glucagon D. IV NaHCO3

Correct Answer - D
Explanation
The treatment of aspirin overdose, also known as salicylate toxicity, typically involves a combination of
supportive care, medications to reduce salicylate levels in the body, and monitoring for complications.

The specific treatments for aspirin overdose may vary depending on the severity of the overdose and the
patient's individual health status, but some common treatments include:

Activated charcoal: This may be given to absorb any remaining aspirin in the stomach and prevent further
absorption into the bloodstream.

Alkalinization of urine: This involves administering intravenous sodium bicarbonate to alkalinize the urine, which
helps to enhance the elimination of salicylates from the body.

Intravenous fluids: These are often given to maintain hydration and electrolyte balance.

Hemodialysis: In severe cases of aspirin overdose, hemodialysis may be required to remove salicylates from the
blood.

27
Monitoring and supportive care: This includes monitoring vital signs, oxygen saturation, and urine output, as
well as treating any complications that may arise, such as respiratory depression or metabolic acidosis.

It is important to note that aspirin overdose can be life-threatening, and prompt medical attention is necessary.
If you suspect an aspirin overdose, seek medical attention immediately or call your local emergency services

Q. 40 - A patient was on haloperidol for psychosis was not responding & given a new drug which causes
adverse effects like increased salivation & dyslipidemia. What is the likely new drug given

A. Risperidone B. Ziprasidone
C. Aripiprazole D. Clozapine

Correct Answer - D
Explanation
Based on the information provided, it is possible that clozapine is the new drug that was given to the patient
who was not responding to haloperidol for psychosis.

Clozapine is an atypical antipsychotic medication that is often used as a second-line treatment for schizophrenia
when other medications have been ineffective. It is known to be effective in treating treatment-resistant
schizophrenia.

However, one of the common side effects of clozapine is increased salivation, which can be bothersome for
some patients. Another potential side effect of clozapine is dyslipidemia, or an abnormal lipid profile, which can
increase the risk of cardiovascular disease.

It is important to note that clozapine is also associated with several other potential side effects, including
agranulocytosis, seizures, myocarditis, and other cardiac complications. Therefore, patients taking clozapine
require close monitoring of their blood counts, heart function, and other vital signs.

If a patient experiences adverse effects from clozapine, their healthcare provider may need to adjust the dose
or switch to a different medication.

28
Q. 41 - A patient was on inhaled steroids for bronchial asthma. He comes to the clinic with oral thrush. Which
antifungal is used for the management of it?

A. Terbinafine B. Flucytosine
C. Clotrimazole D.Griesiofulvion

Correct Answer - C
Explanation
Oral thrush is a common side effect of inhaled corticosteroids used for the treatment of bronchial asthma. It is
caused by an overgrowth of the fungus Candida in the mouth. Clotrimazole is an antifungal medication that can
be used to treat oral thrush.

Clotrimazole is available in various forms, including lozenges, troches, and oral suspensions. It works by
inhibiting the growth and spread of Candida in the mouth.

To use clotrimazole for the management of oral thrush, the patient should follow the instructions provided by
their healthcare provider. This may include using a lozenge or troche several times a day, or using an oral
suspension to rinse the mouth.

It is important to use clotrimazole as directed and continue using it for the full course of treatment, even if
symptoms improve. This will help to ensure that the infection is fully treated and does not return.

In addition to using clotrimazole, it may be helpful for the patient to practice good oral hygiene, such as
brushing and flossing regularly, and avoiding sugary or acidic foods and beverages that can promote the growth
of Candida in the mouth.

Q. 42 - What is the mechanism of action of t PA?

A. Anti thrombin effect B. Fibrinolytic effect


C. Inhibits clot formation D. Anti platelet effect

Correct Answer - B
Explanation
tPA, or tissue plasminogen activator, is a protein that plays a crucial role in the dissolution of blood clots. The
mechanism of action of tPA involves several steps:
29
Activation: tPA is released by endothelial cells in response to injury or other stimuli. It circulates in the blood in
an inactive form until it encounters fibrin, a protein that forms the mesh-like structure of blood clots.

Binding: Once tPA encounters fibrin, it binds to the fibrin molecule, forming a complex.
Conversion: The bound tPA converts plasminogen, an inactive precursor protein, into plasmin, an active
enzyme. Plasmin is responsible for breaking down the fibrin mesh, leading to the dissolution of the clot.

Clearance: tPA is rapidly cleared from the circulation by the liver, reducing the risk of bleeding
complications.Overall, the mechanism of action of tPA involves the conversion of plasminogen to plasmin,
which leads to the dissolution of blood clots. This process is important for restoring blood flow to tissues that
have been compromised by a clot.

Q. 43 - Child with whooping cough & TLC >50000 with absolute lymphocytosis. What is the treatment?

A. Clarithromycin B. Ampicillin
C. Cotrimoxazole D. Azithromycin

Correct Answer - D
Explanation
The treatment for a child with whooping cough (also known as pertussis) and a TLC (total leukocyte count)
>50000 with absolute lymphocytosis involves a combination of antibiotics and supportive care.

The antibiotics of choice for treating pertussis are macrolides such as azithromycin, clarithromycin, or
erythromycin. These antibiotics can reduce the severity and duration of the illness and also decrease the spread
of the bacteria to others.

Supportive care may include measures such as:


● Adequate hydration
● Nutrition
● Oxygen therapy in severe cases
● Monitoring for complications, such as pneumonia or seizures
● Isolation to prevent the spread of the disease

30
Q. 44 - DOC for Gonnorhea?

A. Clarithromycin B. Ampicillin
C.Ceftriaxone D. Azithromycin

Correct Answer - D
Explanation
Ceftriaxone is currently the recommended treatment of choice (DOC) for gonorrhea. This is due to the
increasing prevalence of antibiotic-resistant gonorrhea.
Ceftriaxone is a third-generation cephalosporin antibiotic that is given as an injection into a muscle.

The recommended dose is 250mg. It is usually given along with azithromycin, an oral antibiotic, as dual therapy
to ensure effective treatment. It is important to complete the full course of antibiotics to ensure the infection is
completely cleared.
It is also important to note that sexual partners should be tested and treated if they are found to be infected.

Q. 45 - A patient was on clonidine for hypertension & nicotine replacement therapy. He stopped taking
clonidine & developed headache. What is the mechanism for headache?

A. Receptor hypersensitivity B. Rebound hypertension


C.Hypotension D.Receptor down regulation

Correct Answer - B
Explanation
The abrupt discontinuation of clonidine, a centrally acting alpha-2 adrenergic agonist, can cause rebound
hypertension, which can lead to headaches.

Clonidine works by reducing sympathetic nervous system activity, leading to a decrease in blood pressure.
When it is suddenly discontinued, there is a rapid increase in sympathetic activity, which can cause an increase
in blood pressure and subsequent headaches.

Additionally, nicotine replacement therapy can also cause headaches as a side effect. Nicotine can cause blood
vessels in the brain to constrict, which can lead to headaches.

31
Therefore, the patient in this scenario could be experiencing headaches due to the rebound hypertension
caused by abrupt discontinuation of clonidine and/or as a side effect of nicotine replacement therapy.

Q. 46 - Which of the following statement is true regarding ARB’s.

A. Losartan has TXA2 Antagonism B. Telmisartan undergoes renal clearance


C. Irbesartan undergoes hepatic & renal clearance D. Candesartan undergoes only bile elimination

Correct Answer - A
Explanation
Losartan is an angiotensin II receptor blocker (ARB) and it does not have TXA2 (thromboxane A2) antagonism as
a primary mechanism of action.

Losartan works by selectively blocking the angiotensin II type 1 receptor (AT1) and thereby inhibiting the
vasoconstrictor and aldosterone-secreting effects of angiotensin II.

This leads to vasodilation, a decrease in blood pressure, and a reduction in sodium and water retention.

32
Forensic Medicine

Q. 47 - A sample from a perineal area of 10 year old. A test was performed using picric acid that showed
yellow needle shaped crystals. What is the test?

A. Barberio’s test B. Acid phosphatase test


C. Florence test D. Tiechman’s test

Correct Answer - A
Explanation
Test for spermine that has been used in the past is called the Barberio test.
Barberio’s test was invented by Barberio in the year 1905.

When the stain is allowed to react with picric acid it leads to the formation of yellow needle shaped spermine
picrate crystals, including the presence of seminal stain.

Q. 48 - The technique of autopsy that is done to limit the spread of infection in HIV Patients

A. Rokitansky technique B. Virchow technique


C. Letulle technique D. Ghon’s technique

Correct Answer - A
Explanation
The Rokitansky technique, also known as the "classical autopsy," is a standardized method of conducting
autopsies that was developed by Carl Rokitansky, a pathologist from Vienna, in the mid-19th century.

This technique involves a systematic examination of all organs and tissues in the body to identify the cause and
manner of death.

The Rokitansky technique begins with a careful external examination of the body, including documentation of
any injuries or abnormalities.
The next step is to perform an internal examination of the organs and tissues in the body.

33
This involves making a Y-shaped incision on the chest and abdomen and removing the organs in a specific order,
starting with the lungs and ending with the brain.

During the organ removal process, the organs are carefully examined and weighed, and samples of tissues are
taken for microscopic analysis. The organs are then examined further to identify any abnormalities, such as
tumors or infections. Finally, the organs are returned to the body and the incisions are closed.

The Rokitansky technique is still widely used today, although some modifications may be made depending on
the specific circumstances of the autopsy.

While this technique is not specifically designed to limit the spread of infection in HIV patients during autopsies,
special precautions must be taken to prevent the transmission of infectious diseases, as outlined by established
guidelines and protocols

Q. 49 - In a dead body temperature was found to be 39 deg celsius. What is the most likely cause of death?

A. Cyanide poisoning B. Corrosive poisoning


C. Septicemia D. Intra abdominal hemorrhage

Correct Answer - C
Explanation
In a deceased individual, the features of septicemia may be more difficult to determine, as some of the classic
signs and symptoms of septicemia, such as fever or hypotension, may no longer be present. However, there are
some indications that may suggest septicemia as a possible cause of death, such as:

Evidence of multiple organ failure: Examination of the organs during autopsy may reveal signs of damage or
dysfunction in multiple organ systems, which is a common feature of septicemia.

Disseminated intravascular coagulation (DIC): DIC can cause small blood clots to form throughout the body,
which can be seen during autopsy as evidence of bleeding or ischemia (lack of blood flow) in various tissues.

Evidence of infection: Septicemia is a type of infection that can arise from a variety of sources, such as
pneumonia, urinary tract infections, or skin infections. Autopsy findings may reveal evidence of infection in one
or more organs or tissues.

34
Presence of bacteria in the blood or tissues: Blood cultures and tissue samples taken during autopsy may reveal
the presence of bacteria or other microorganisms that are associated with septicemia.

Q. 50 -The technique of autopsy that employs the en block method of removing thoracic, cervical &
abdominal organs?

A. Rokitansky technique B. Virchow technique


C. Lettuce technique D. Ghon’s technique

Correct Answer - D
Explanation
Ghon's technique of autopsy, also known as the Ghon method, is a method of performing a post-mortem
examination of the lungs. It was developed by Anton Ghon, an Austrian pathologist, in the early 20th century.

The Ghon method involves dissecting the lungs in a systematic way, examining each lobe individually. The lungs
are first removed from the chest cavity and weighed, then examined for any external abnormalities such as
scars or adhesions. Next, the lung tissue is cut into sections and examined under a microscope for signs of
disease or damage.

The Ghon method is particularly useful in diagnosing tuberculosis, as it allows the pathologist to identify the
characteristic lesions, or Ghon foci, that are present in the lungs of individuals with this disease.

In addition to its diagnostic applications, the Ghon method has been used to study the pathology of a wide
range of lung diseases, including pneumonia, lung cancer, and cystic fibrosis.

It remains an important tool in modern pathology and is often used in conjunction with other diagnostic
techniques such as imaging studies and laboratory tests.

Q. 51 - Sec 314 of IPC deals with

A. Criminal abortion with consent B. Criminal abortion without consent


C. Criminal abortion leading to death of mother D. Preventing a child from being born alive

35
Correct Answer - B
Explanation
Section 314 of the Indian Penal Code (IPC) deals with the offense of "Death caused by act done with the intent
to cause miscarriage, if act done without woman's consent".

According to this section, whoever, with the intention of causing the miscarriage of a woman, does any act
which causes the death of such woman, shall be punished with imprisonment for life, or with imprisonment of
either description which may extend to ten years, and shall also be liable to fine.

It is important to note that this section only applies if the act causing the miscarriage was done without the
woman's consent. If the act was done with the woman's consent, it may still be punishable under other
provisions of the IPC, such as Section 313 which deals with causing miscarriage without the woman's consent.

Section 314 is a serious offence, as it involves causing the death of a woman with the intent to cause
miscarriage. The section is aimed at preventing and punishing acts that are harmful to the life and health of
women, and protecting their right to make decisions about their own bodies

Q. 52 - During harvest season, a farmer was taking rest in shade. He felt something biting & woke up to see it
moving away quickly. He felt drowsy, pain & uncontrolled bleeding. The wound becomes red with blisters.
Which of the following is the possible cause?

A. Scorpion B. Viper
C. Wasp D. Krait

Correct Answer - B
Explanation
It is possible that the farmer was bitten by a viper and is experiencing symptoms of venom poisoning. Vipers are
venomous snakes that can cause severe symptoms such as drowsiness, pain, uncontrolled bleeding, and
blistering of the skin. If left untreated, viper bites can be fatal.

It is important to seek medical attention immediately if you suspect that you have been bitten by a venomous
snake, as prompt treatment can be life-saving. In the meantime, keep the affected limb immobilized and at or
below heart level to slow the spread of venom. Avoid any unnecessary movement, and do not try to suck out
the venom or apply a tourniquet, as these can worsen the situation.

36
Q. 53 - A pt. Presented with seizures, m/s tremor, backward aching of spine after consuming some herbal
medicine. What is the most likely cause?

A. Strychnine poisoning B. Oleander poisoning


C. Aconite poisoning D. Castor seed poisoning

Correct Answer - A
Explanation
Strychnine poisoning can cause several symptoms, including seizures, muscle stiffness and tremors, and back

arching. It is important to note that these symptoms can also be caused by other conditions or toxins, so a

thorough evaluation and diagnosis is necessary.

If a patient presents with these symptoms after consuming herbal medicine, it is important to investigate the

ingredients of the herbal medicine to determine if strychnine or other potentially toxic substances were

present. Treatment for strychnine poisoning may involve supportive measures such as respiratory support and

intravenous fluids, as well as medications such as benzodiazepines to control seizures and muscle stiffness.

It is important to seek medical attention immediately if someone is experiencing these symptoms, as strychnine

poisoning can be life-threatening.

37
Pathology

Q. 54 - A 35 year old male patient came with C/O fatigue & weakness. On investigations peripheral smear
showed below finding. Which is most likely genetic change associated with the patient?

A. T(9,22) B. T (15,17)
C. T (11, 14) D. T(14, 18)

Correct Answer - B
Explanation
The image shows Auer cells
Bundles of Auer rods aggregate to form Faggot cell that is, pathognomonic of AML-M3
Translocation between chromosomes 15 and 17 (seen most often in patients with M3)

Q. 55 - A 44 year old person presented with C/O lymphadenopathy. O/E he had generalized
lymphadenopathy and no organomegaly. Peripheral smear analysis showed lymphocytosis and smudge cells.
Which of the following is the next step in diagnosis?

A. Lymph node biopsy B. Flow cytometry


C. BM examination D. Genetic analysis

Correct Answer - B
Explanation
Flow cytometry is a technology that rapidly analyzes single cells or particles as they flow past single or multiple
lasers while suspended in a buffered salt-based solution. Each particle is analyzed for visible light scatter and
one or multiple fluorescence parameters.

38
Q. 56 - A 7 year old kid came to OPD with C/O swelling in the jaw. O/E she was found to have cytopenias and
BM aspiration shown below cells. Most likely diagnosis is?

A. Follicular lymphoma B. Adult T cell leukemia


C. Burkitt's lymphoma D. Infectious mononucleosis

Correct Answer - C
Explanation
Burkitt lymphoma (BL) is an aggressive non-Hodgkin B-cell lymphoma.
The t(8;14)(q24;q32) is the most common translocation in BL
Burkitt lymphoma is an aggressive B-cell lymphoma comprising a monomorphic population of
intermediate-sized mature lymphocytes.

"starry sky" appearance due to tingible-body macrophages containing cellular debris


Patients present with abdominal pain secondary to ileocecal disease, abdominal distention, nausea, vomiting,
and gastrointestinal bleeding. Adult patients are more likely to present with constitutional symptoms (i.e., fever,
weight loss, night sweats)

Q. 57 - Identify the structure in the below histology slide?

A. Islet of langerhans B. Glomerulus


C. Lymph node D. Hassall corpuscles

39
Correct Answer - B
Explanation
The glomerulus is a complex web of capillaries derived from the afferent arteriole.

The glomerulus is essentially a small tuft of capillaries, known as the glomerular capillaries, surrounded by a
several layers of epithelial cells which provide a selective filtration barrier and direct filtered fluids into the
remainder of the nephron.

Q. 58 - Which of the following statements are true regarding relative risk of Mendelian pattern of
inheritance?

A. 2/3 autosomal dominant B. 1/4 autosomal recessive


C. 1/10 Y linked D. 1/5 X linked recessive

Correct Answer - B
Explanation
There are five basic modes of inheritance for single-gene diseases: autosomal dominant, autosomal recessive,
X-linked dominant, X-linked recessive, and mitochondrial.

The relative risk of autosomal recessive inheritance also depends on the specific disorder. In autosomal
recessive inheritance, an affected individual must inherit two copies of the disease-causing allele (one from
each parent) to manifest the disorder. In general, the risk of two carrier parents having an affected child is 1 in
4, or 25%.

Q. 59 - A 5 year old kid came with a history of fever and he was diagnosed to have ventricular
tachycardia and dysfunction. Below is the HPE of the patient's heart. Likely diagnosis is?

40
A. Chaga’s disease B. Rheumatic heart disease
C. Lymphocytic myocarditis D. pericarditis

Correct Answer - C
Explanation
Myocarditis is an inflammatory disease of the myocardium that is an established cause of sudden death for
both adult and pediatric populations.

Myocarditis can present as nonspecific symptoms, such as shortness of breath, nausea/vomiting, and
tachycardia, but it can also mimic acute coronary syndrome.
The most common causes of death due to myocarditis are heart failure and dysrhythmia.

Lymphocytic myocarditis. This intermediate magnification of a myocardium shows diffuse infiltrates composed
mostly of lymphocytes but also some neutrophils, macrophages, and plasma cells.

Q. 60 - A 34 year old man working in the cement industry came wit history of breathing difficulty. O/E there
was opacity on chest X-ray and was confirmed histologically to have mesothelioma. Which of the following is
most likely cause?

A. Silicosis B. Asbestosis
C. Coal D. Iron oxide

Correct Answer - B
Explanation
Asbestosis is an interstitial lung disease caused by the inhalation of asbestos fibers.
There are three main forms of asbestos exposure. Direct work-related environmental exposure is common
among workers at shipyard, mining, aerospace, etc.

Bystander exposure is the second form of exposure commonly seen in certain professionals like electricians,
masons, and painters.

The third and most common form of asbestos exposure is the general community exposure, such as the use of
asbestos for road surfaces, playground material, landfills, and chemical paints.

41
Interstitial fibrosis is regarded as the principal pathogenic mechanism of asbestosis.
Asbestos bodies are golden yellow, beaded rod-like structures with a translucent center.

It is formed when asbestos fibers get coated with iron-containing proteinaceous material
History of smoking and dyspnea on exertion are important, and nonproductive cough is quite common.

History of loss of appetite and weight, hemoptysis indicates the suspicion of lung tumors. Gradual onset of
localized pain or breathlessness and then radiating to the shoulder may be evident in pleural involvement

Q. 61 - A 20 year old with history of pedal edema and on investigation was found to have proteinuria. On
kidney biopsy there was a spike and dome appearance. Most likely diagnosis is?

A. Minimal change disease B. Membranous glomerulonephritis


C. FSGS D. C3 glomerulonephritis

Correct Answer - B
Explanation
Membranous nephropathy (MN) is an immune complex deposition disease.
Up to 80% of patients with MN present with nephrotic range proteinuria with preserved renal function. RBC
casts are rare but microscopic hematuria is common (up to 40%).

By light microscopy, the basement membrane is observed to be diffusely thickened. Using Jones' stain, the GBM
appears to have a "spiked" or "holey" appearance.

Q. 62 - A 12 year old with a history of pyoderma 2 weeks back was investigated. She had hematuria &
creatinine levels was 2 mg%. 3 weeks later follow up her creatinine went up 4.5mg%. Which of the following
will be the EM finding in a patient?

A. Subepithelial deposit B. Subendothelial deposit


C. Crescents D. Intramembranous deposit

42
Correct Answer - A
Explanation
IgA nephropathy (IgAN) is defined by the presence of dominant or co-dominant mesangial IgA immune
deposits, often accompanied by C3 deposits and abnormal findings on urinalysis

The light microscopic features of IgAN are diverse; mesangial lesions vary from minimally changed to diffusely
proliferated, and may have sclerosis and/or crescents

43
ENT
Q. 63 - Patient has negative rinne on 256 and 512 Hz tunning fork but positive rinne on 1024 Hz. What is the
air bone gap?

A. 15 to 30 Db B. 30 to 45 Db
C. 45 to 60 Db D. >60 Db

Correct Answer - B
Explanation
Rinne +ve = AC>BC (If Rinner +Ve it can be normal or SNHL)
So, AC will be longer & lounder
If Rinner -ve = AC < BC (Rinne can be CHL)
Ideally, 3 tuning forks are used 256, 512 and 1024 Hz.
Rinne’s tuning fork test can be used to quantify the degree of conductive hearing loss into mild (20-30 dB),
moderate (30-45 dB), and severe (45-60 dB).
Rinne -ve 256, 512
Rinne +ve 1024 --> air-bone gap range of 30- 45dB

Q. 64 - Identify the marked structure

A. Tubal tonsil B. Fossa of Rosenmuller


C. Eustachian tube opening D. Torus tubarius

Correct Answer - A
Explanation
The tubal tonsil is one of the four main tonsil groups comprising Waldeyer's tonsillar ring, which includes the
Palatine tonsil, the Lingual tonsil, thePharyngeal tonsil, and the Tubal tonsil.
It is located posterior to the opening of the Eustachian tube into the nasopharynx.

44
Q. 65 - A old patient presented with change in voice. He was diagnosed with a cancer and underwent an
intervention. Following which surgery is the following intervention done?

A. Partial laryngectomy B. Total Laryngectomy


C. Vertical Hemilaryngectomy D. Horizontal hemilaryngectomy

Correct Answer - B
Explanation
A standard total laryngectomy procedure encompasses the entire larynx with its attached prelaryngeal strap
muscles and the lymph nodes in the jugular chain (levels II, III, and IV).

The most common skin incision used for total laryngectomy is the Gluck - Sorenson U-shaped incision.

The starting point of the incision is located at the apex of the mastoid, continued on the anterior border of the
sternocleidomastoid muscle up to 1 to 2 cm above the upper edge of the sternal notch, and it is continued in
the same way on the opposite side.

Q. 66 - Which of the following is not an indication for the following device?

45
A. Pulmonary toilet B. upper airway assessment
C. Acute upper airway obstruction D. Prolonged mechanical ventilation

Correct Answer - B
Explanation
Tracheostomy tube
Upper airway assessment is not an indication for a tracheostomy tube. Thank you for pointing that out.

The indications for a tracheostomy tube, are related to situations where the normal airway is compromised or
needs to be bypassed.

These indications can include:

1. Prolonged Mechanical Ventilation: When a patient requires long-term or chronic mechanical


ventilation, a tracheostomy tube may be inserted to provide a secure airway access and facilitate
ventilatory support.

2. Upper Airway Obstruction: In cases where there is severe upper airway obstruction, such as due
to tumors, trauma, or swelling, a tracheostomy tube can be used to establish a stable airway
below the obstruction.

3. Failure to Wean from Ventilator: Some patients may have difficulty weaning off mechanical
ventilation through an endotracheal tube. In such cases, a tracheostomy tube may be considered
to facilitate weaning and reduce the risk of complications associated with prolonged
endotracheal intubation.

4. Respiratory Distress or Failure: If a patient is experiencing severe respiratory distress or


respiratory failure and other interventions have been ineffective, a tracheostomy tube can
provide a more secure airway and improve oxygenation and ventilation.

5. Long-term Airway Access: In certain medical conditions or situations, such as severe neurological
impairment or an anticipated need for prolonged ventilation, a tracheostomy tube may be
recommended for long-term airway management and support.

46
Ophthalmology

Q. 67 - An infant difficulty in opening the eye present to you with cloudy cornea. There is also watering from
baby’s eye since birth. What is the most probable diagnosis looking at the picture given

A. Congenital glaucoma B.Mucopolysaccharoidosis


C. Congenital cataract D. Opthalmia neonatarum

Correct Answer - A
Explanation
Primary congenital glaucoma (PCG) is a rare disease due to genetically-determined abnormalities in the
trabecular meshwork and anterior chamber angle resulting in elevated intraocular pressure (IOP), without other
ocular or systemic developmental anomalies.

It occurs when the eye hasn't developed properly in the womb and this leads to issues with the flow of aqueous
fluid out of the eye. If the fluid is unable to drain effectively, it builds up inside the eye and causes the eye
pressure to rise.

Q. 68 - A 65 year old male presented with colored haloes & gradual decrease in vision. There is a h/o DM
since many years. What is the diagnosis & management of the patient looking at the slit lamp picture.

A. Sunflower cataract/ snow flake cataract- control sugar levels


B. Zonular cataract- lens extraction
C. Immature senile cataract- Phacoemulsification & IMSC
D. Rossette cataract- ECCE

47
Correct Answer - C
Explanation
Immature senile cataract refers to a type of cataract that is still developing and has not yet reached its full
maturity. Phacoemulsification and intracapsular manual small incision cataract surgery (IMSC) are two surgical
options for treating this condition.

Phacoemulsification is a minimally invasive surgical technique used to remove cataracts from the eye. It involves
the use of a small probe that emits high-frequency sound waves to break up the cloudy lens into small pieces.
These pieces are then removed through a small incision in the eye, and a new artificial lens is implanted in its
place.

IMSC, on the other hand, involves the manual removal of the entire lens, along with the capsule that surrounds
it, through a small incision in the eye. This technique is typically used in cases where phacoemulsification is not
possible due to factors such as a very hard or mature cataract.

Both phacoemulsification and IMSC are highly effective at treating immature senile cataracts, with a very low
risk of complications. Your eye surgeon will be able to recommend the best option for your individual case
based on your eye health, age, and other factors

Q. 69 - Which is the best diagnostic modality for this corneal condition

A. Rose bengal dye B. Lissamine green stain


C. Fluorescein dye with cobalt blue light D. Fluorescein dye with green filter

Correct Answer - C
Explanation
Fluorescein dye with cobalt blue light is a diagnostic technique used to examine the surface of the eye and
detect abnormalities. Fluorescein is a yellow-green fluorescent dye that is injected into a vein in the arm, and it
quickly circulates to the blood vessels in the eye. The cobalt blue filter is then used to view the dye as it
fluoresces, or emits light, under certain wavelengths.
48
The dye highlights any areas of damage or abnormality on the surface of the
eye, including corneal abrasions, ulcers, foreign bodies, and other conditions. The cobalt blue filter enhances
the visibility of the fluorescein dye, making it easier for the eye doctor to detect any abnormalities.

This diagnostic technique is safe and painless, with few risks or side effects. It is commonly used in eye exams
and can help diagnose and monitor a variety of eye conditions, including dry eye syndrome, corneal
dystrophies, and corneal infections. It is also used in contact lens fitting to assess the fit and health of the
contact lens on the eye.

Q. 70 - A 65 year old pt. Came with binasal vision loss. Loss of vision on left side. There is space occupying
lesion & he is on some treatment. What is the most probable location of the lesion.

A. Chiasma B. Rt. Optic tract


C. Lt. Optic tract D. Lt. Visual cortex

Correct Answer - B
Explanation
If the lesion is located on the right optic tract, it would affect the visual signals coming from the left eye as the
right optic tract carries information from the left visual field. This would result in loss of vision on the right side
(right homonymous hemianopia) of both eyes.

Lesions of the optic tract can be caused by a variety of conditions, including stroke, tumors, and demyelinating
diseases such as multiple sclerosis. Treatment of the lesion will depend on the underlying cause, and may
involve medication, surgery or radiation therapy.

Q. 71 - What is the refractive error shown in this image of sturm's conoid.

49
A. Hypermetropia B. Myopia
C. Astigmatism D. Presbyopia

Correct Answer - C
Explanation
Sturm's conoid is a three-dimensional representation of astigmatism. It is a conical surface that is generated by
plotting the refraction of light rays passing through a cylindrically shaped lens.

The curved lines represent the refractive power of the cylindrical lens at different points across the lens surface.
The center of the conoid represents the point of zero astigmatism, while the outer edges represent the points
of maximum astigmatism.

This image can be used to help understand and diagnose astigmatism, a common refractive error that causes
blurred vision at all distances. Astigmatism occurs when the cornea or lens of the eye has an irregular shape,
causing light rays to be refracted unevenly and focus at multiple points on the retina. Sturm's conoid provides a
visual representation of how this uneven refraction occurs in cylindrical lenses, and can help eye care
professionals design corrective lenses to compensate for the astigmatism.

Q. 72 - A child came with this dark colored expanding lesion on one side of the face since birth. What is the
most likely long term complication for him in future.

50
A. Cataract B. Glaucoma
C. Malignant melanoma D. Loss of vision

Correct Answer - B
Explanation
Nevus of Ota is a rare, non-cancerous condition that affects the eye and skin around the eye. It is caused by an
overproduction of melanin in the skin and the eye. While Nevus of Ota does not usually cause any serious
health problems, it can increase the risk of certain complications.

One of the potential long-term complications of Nevus of Ota is glaucoma. Glaucoma is a group of eye
conditions that can cause damage to the optic nerve and lead to vision loss. In people with Nevus of Ota, the
increased melanin in the eye can cause a buildup of pressure, which can damage the optic nerve and increase
the risk of developing glaucoma.

However, it is important to note that not everyone with Nevus of Ota will develop glaucoma, and the risk of
developing glaucoma can vary depending on a number of factors, such as the size and location of the nevus,
and the age of the person.

Q. 73 - A 2-year-old child came with mental retardation & a cherry red spot in retina. What is the most likely
diagnosis.

A. Tay sach disease B. Hurler disease


C. Hunter disease D. Neiman pick disease

Correct Answer - A
Explanation
Tay-Sachs disease is a rare genetic disorder that primarily affects the nervous system.
It is caused by the deficiency of an enzyme called hexosaminidase A, which leads to the accumulation of a fatty
substance called GM2 ganglioside in the nerve cells.

Clinical features of Tay-Sachs disease:

Onset and Progression: Tay-Sachs disease is usually diagnosed in infancy, and affected infants typically appear
healthy until around 3 to 6 months of age. After that, they start to show developmental delays and progressive

51
loss of motor skills, such as crawling and sitting. They may also develop seizures, blindness, and hearing loss.
Tay-Sachs disease is usually fatal, and affected children usually die by the age of 4 or 5.

Neurological Symptoms: Tay-Sachs disease primarily affects the nervous system, leading to a range of
neurological symptoms, including muscle weakness, decreased muscle tone (hypotonia), and exaggerated
reflexes (hyperreflexia). As the disease progresses, the child may become unresponsive and develop spasticity,
which is characterized by stiff, jerky movements.

Cherry-Red Spot: One of the characteristic features of Tay-Sachs disease is the appearance of a cherry-red spot
in the center of the retina, which can be seen during an eye exam. This is caused by the accumulation of GM2
ganglioside in the cells of the retina.

Inheritance: Tay-Sachs disease is inherited in an autosomal recessive pattern, which means that a child must
inherit two copies of the defective gene (one from each parent) to develop the disease. If both parents are
carriers of the Tay-Sachs gene, there is a 25% chance with each pregnancy that their child will inherit two copies
of the defective gene and develop Tay-Sachs disease.

There is currently no cure for Tay-Sachs disease, and treatment focuses on managing the symptoms and
providing supportive care. Genetic counseling and carrier screening are recommended for couples with a family
history of the disease or who are at risk of being carriers.

Q. 74 - A patient came with H/o dementia & noticed to have a golden yellow ring around cornea. Which of
the following investigation will help you in diagnosis.

A. Sr. ceruloplasmin level B. Sr. calcium level


C. Alpha-fetoprotein level D. Sr. iron level

Correct Answer - A
Explanation
The combination of dementia and a golden yellow ring around the cornea suggests the possibility of Wilson's
disease, which is a rare genetic disorder that causes copper to accumulate in various organs, including the brain
and liver.

52
The golden yellow ring around the cornea is known as a Kayser-Fleischer ring, which is caused by the deposition
of copper in the Descemet's membrane of the cornea. The presence of Kayser-Fleischer rings is a characteristic
feature of Wilson's disease.

Serum ceruloplasmin level is a blood test that measures the level of ceruloplasmin, which is a copper-binding
protein. In Wilson's disease, the level of ceruloplasmin is usually low due to impaired copper metabolism, and
this finding can help in the diagnosis of the disease.

However, it is important to note that a low ceruloplasmin level can also be caused by other conditions, such as
malnutrition or chronic liver disease, so additional testing may be necessary to confirm the diagnosis of
Wilson's disease.
Other diagnostic tests for Wilson's disease include liver function tests, urine copper levels, and genetic testing.

Q. 75 - There is a child with swelling in left eye. Diagnosis of condition

A. Dermoid of limbus B. Lipodermoid


C. Pingecula D. Pterygium

Correct Answer - A
Explanation
A dermoid of the limbus, also known as a limbal dermoid, is a type of benign tumor that occurs at the junction
of the cornea and sclera in the eye. It is typically present at birth, although it may not be noticed until later in
life. Limbal dermoids are usually located on the nasal side of the limbus and appear as a raised, yellowish-white
mass with hair follicles and skin appendages.

Limbal dermoids are thought to arise from a developmental abnormality in which skin cells are trapped in the
eye during embryonic development. They are typically unilateral, although they can occur in both eyes in rare
cases. Limbal dermoids are usually asymptomatic and do not affect vision, although they can cause irritation
and inflammation if they become infected or inflamed.
Diagnosis include visual acuity testing, slit-lamp biomicroscopy, and possibly ultrasound imaging.
53
Treatment may not be necessary if the dermoid is small and asymptomatic, but larger or symptomatic dermoids
may require surgical removal.

In some cases, a limbal dermoid may be associated with other ocular abnormalities, and additional testing or
evaluation may be necessary to rule out other underlying conditions.

Q. 76 - A patient with a gradual decrease of vision also now coming to you with myopia. What is the most
likely cause of his refractory condition.

A. Axial myopia B. Index myopia


C. Positional myopia D. Curvatural myopia

Correct Answer - B
Explanation
If a patient presents with a gradual decrease in vision and is diagnosed with myopia, it is important to
determine whether the myopia is due to index myopia or another type of myopia, such as axial myopia or
curvature myopia.

Index myopia is caused by a refractive error in the cornea and lens of the eye, which results in light focusing in
front of the retina instead of on it. This type of myopia can be corrected with glasses or contact lenses that
adjust the refractive power of the eye to allow light to focus correctly on the retina.

Axial myopia, on the other hand, is caused by the eyeball being too long, which causes light to focus in front of
the retina. This type of myopia can also be corrected with glasses or contact lenses, but may also require
additional treatment such as orthokeratology or refractive surgery to reshape the cornea.

Curvature myopia is caused by an abnormal curvature of the cornea, which causes light to focus in front of the
retina. This type of myopia can also be corrected with glasses or contact lenses, but may also require additional
treatment such as corneal reshaping surgery.

To determine the cause of a patient's myopia, an eye exam will be necessary. This may include visual acuity
testing, a refraction test to measure the eye's refractive error, and possibly imaging tests such as an ultrasound
or optical coherence tomography (OCT) to assess the shape and length of the eyeball.
54
Preventive & Social Medicine

Q. 77 - A 10-year-old child was presented to OPD. Which vaccine would you advise according to the national
immunization schedule

A. Td vaccine B. Typhoid vaccine


C. BCG D. DPT

Correct Answer - A
Explanation
Td is usually given as a booster dose every 10 years, or after 5 years in the case of a severe or dirty wound or
burn.

Another vaccine, called “Tdap,” may be used instead of Td. Tdap protects against pertussis, also known as
“whooping cough,” in addition to tetanus and diphtheria.

Age Vaccines given

Birth Bacillus Calmette Guerin (BCG), Oral Polio Vaccine (OPV)-0 dose,
Hepatitis B birth dose

6 Weeks OPV-1, Pentavalent-1, Rotavirus Vaccine (RVV)-1, Fractional dose of


Inactivated Polio Vaccine (fIPV)-1, Pneumococcal Conjugate Vaccine
(PCV) -1*

10 weeks OPV-2, Pentavalent-2, RVV-2

14 weeks OPV-3, Pentavalent-3, fIPV-2, RVV-3, PCV-2*

9-12 Measles & Rubella (MR)-1, JE-1** , PCV-Booster*


months

16-24 MR-2, JE-2**, Diphtheria, Pertussis & Tetanus (DPT)-Booster-1, OPV –


months Booster

5-6 years DPT-Booster-2

10 years Tetanus & adult Diphtheria (Td)

16 years Td

Pregnant Td-1, Td-2 or Td-Booster***


Mother

55
Q. 78 - A 22-year-pregnant woman presented to the subcentre with h/o eating papaya. She suspects that she
might have aborted the child. You have examined the patient thoroughly & the pregnancy is safe & ask her to
go back home. Which health worker will be sent for rural visit for this kind of situation?

A. ASHA B. Train birth attendant


C. Anganwadi worker D. MO

Correct Answer - C
Explanation
The anganwadi workers are responsible for carrying out certain important functions which are listed as follows:

∙ Assist the doctors in the campaigns for immunization.


∙ Create awareness related to reproduction.
∙ Provide drugs and medicines kit to the needy.
∙ Keeping records of immunization programs and the people involved .
∙ Reporting the cases of diseases.
∙ Creating awareness about the hygiene, diseases, etc.
∙ Conducting preschool activities.

Q. 79 - A study among 2 groups of people working in an aniline dye factory for 20 years with people working
in factory & office worker of same factory ( saw the document). What type of study is this?

A. Case control study B. Retrospective cohort study


C. Prospective cohort study D. Ecological study

Correct Answer - B
Explanation
A retrospective cohort study, also called a historic cohort study, is a longitudinal cohort study used in medical
and psychological research. A cohort of individuals that share a common exposure factor is compared with
another group of equivalent individuals not exposed to that factor, to determine the factor's influence on the
incidence of a condition such as a disease or death.

Retrospective cohort studies have existed for approximately as long as prospective cohort studies

56
The retrospective cohort study compares groups of individuals who are alike in many ways but differ by a
certain characteristic (for example, female nurses who smoke and ones who do not smoke) in terms of a
particular outcome (such as lung cancer).

Data on the relevant events for each individual (the form and time of exposure to a factor, the latent period,
and the time of any subsequent occurrence of the outcome) are collected from existing records and can
immediately be analyzed to determine the relative risk of the cohort compared to the control group.

Q. 80 - How to collect tap water for bacteriological study


A. Before collecting water let water flow for 1 min
B. Attach bottle before turning on the tap to avoid splashing
C. Attach bottle to mid stream & avoid splashing
D. Tap water from leaking tap

Correct Answer - A
Explanation
Bacteriological water testing is a process that involves analyzing water samples to determine the presence and
concentration of bacteria. This testing is crucial for ensuring the safety and quality of drinking water, as bacteria
can cause various waterborne diseases and contamination issues.

Open a cold water fixture to a wide-open setting and allow the water to run a minimum of five minutes.
Steps:-
Sample Collection: Water samples are collected from various sources such as faucets, wells, rivers, lakes, or any
other potential water supply. Proper sterilized containers are used to collect the samples, ensuring they are free
from external contamination.

Transport and Preservation: After collection, the water samples need to be transported to the laboratory for
testing. During transportation, it's important to keep the samples at an appropriate temperature and avoid any
potential changes that could affect the bacterial content.

Preparation: In the laboratory, the water samples are prepared for testing. This may involve filtration or dilution
of the samples to obtain an appropriate concentration for analysis.

Bacterial Enumeration: The most common method used for bacteriological water testing is the Membrane
Filtration Technique. In this method, a measured volume of the water sample is passed through a membrane
57
filter that retains the bacteria present. The filter is then placed on a nutrient agar medium that promotes
bacterial growth. After incubation, the colonies that develop on the filter are counted and expressed as
colony-forming units (CFUs) per unit volume of water.

Identification: Once the colonies have been enumerated, further tests can be conducted to identify specific
types of bacteria present in the water sample. This may involve biochemical tests, molecular techniques (such
as PCR), or other specialized methods depending on the requirements.

Interpretation and Reporting: The results of the bacteriological water testing are interpreted and compared to
the regulatory standards or guidelines established by the local authorities. A comprehensive report is
generated, indicating the bacterial content, any potential contamination issues, and the overall quality of the
water sample.

Q. 81 - What is the best measure to assess the health status of a community, a sensitive indicator of
availability & utilization of health services
A. IMR B. MMR
C. DALY D. HDI

Correct Answer -A
Explanation
The infant mortality rate is the number of infant deaths for every 1,000 live births.

Infant mortality represents an important component of under-five mortality. Like under-five mortality, infant
mortality rates measure child survival. They also reflect the social, economic and environmental conditions in
which children (and others in society) live, including their health care.

Q. 82 - Which of the statement is correct about the Colombo plan

A. Chemotherapy B. Cancer units


C. Cobalt supply D. Manpower strengthening

Correct Answer - C
Explanation
The Colombo Plan is a partnership concept of self-help and mutual-help in development aimed at
socio-economic progress of its member countries.
Under the Colombo Plan canada supply Cobalt therapy units to medicals established in india.
58
Q. 83 - A 10 year old child presented to pediatric opd with fever & rashes & koplik spot on buccal mucosa.
Identify the diagnosis?

A. Measles B. Mumps
C. Rubella D. Chicken pox

Correct Answer - A
Explanation
Measles is a highly contagious viral infection caused by the measles virus. It primarily affects children but can
occur in individuals of any age who are not immune. Here are some key features and characteristics of measles:

Transmission: Measles is primarily transmitted through respiratory droplets when an infected person coughs or
sneezes. The virus can remain in the air or on surfaces for up to two hours, making it highly contagious.

Incubation Period: The incubation period for measles is typically around 10 to 12 days, during which the person
does not exhibit any symptoms.

Prodromal Phase: Measles begins with a prodromal phase, which lasts for 2 to 4 days. During this phase, the
infected individual may experience symptoms such as high fever, malaise (generalized discomfort or
uneasiness), loss of appetite, tiredness, and a dry cough.

Three Cs: Measles is often associated with the "Three Cs" of symptoms: Cough, Coryza (nasal congestion), and
Conjunctivitis (red, watery eyes). These symptoms typically become more prominent during the prodromal
phase.

Koplik's Spots: Koplik's spots are small, white spots that appear on the inside of the cheeks and are
characteristic of measles. They typically appear 1 to 2 days before the onset of the rash.

Measles Rash: The measles rash is a distinct feature of the disease. It typically starts on the face and gradually
spreads downward to the trunk, arms, and legs. The rash consists of flat, red or reddish-brown spots that may
merge together. It usually lasts for about 5 to 6 days and gradually fades.

High Fever: Measles is often accompanied by a high fever, which can spike to 104°F (40°C) or even higher.

59
Complications: Measles can lead to various complications, especially in young children and individuals with
weakened immune systems. These complications may include ear infections, pneumonia, encephalitis (swelling
of the brain), and in severe cases, death.

Immunity: Once a person has had measles, they develop lifelong immunity to the virus. However, it's still
possible to contract other strains of the measles virus.

Vaccination: Vaccination is the most effective way to prevent measles. The measles, mumps, and rubella (MMR)
vaccine is routinely administered to children as part of the recommended vaccination schedule

Q. 84 - Identify the vitamin deficiency in child whose image is given below

A. Vit D Deficiency B. Vit A Deficiency


C. Vit K Deficiency D. Vit E Deficiency

Correct Answer - A
Explanation
Vitamin D deficiency occurs when the body does not receive an adequate amount of vitamin D, either from
dietary sources or sunlight exposure. Here are some features and symptoms associated with vitamin D
deficiency:

Bone and Muscle Weakness: Vitamin D plays a crucial role in calcium absorption and bone health. Insufficient
levels of vitamin D can lead to weak bones and muscles, which can result in conditions like osteomalacia
(softening of bones) in adults and rickets (softening and weakening of bones) in children.

Bone Pain: Vitamin D deficiency can cause bone pain, especially in the lower back, hips, and legs. This pain may
be dull, achy, or throbbing in nature.

60
Muscle Pain: Muscular weakness and pain can occur due to vitamin D deficiency. Individuals may experience
generalized muscle aches or weakness, which can affect their mobility and physical performance.

Increased Susceptibility to Fractures: Insufficient vitamin D levels can compromise bone strength and density,
increasing the risk of fractures, particularly in the elderly.

Fatigue and Weakness: Vitamin D deficiency is associated with feelings of fatigue, tiredness, and overall
weakness. Individuals may experience reduced energy levels and a general lack of stamina.

Impaired Wound Healing: Vitamin D is involved in the regulation of the immune system and the healing process.
Deficiency in vitamin D may impair wound healing and delay the recovery from injuries or surgeries.

Mood Changes and Depression: Some studies suggest a link between vitamin D deficiency and mood disorders
such as depression. While the exact relationship is not fully understood, low levels of vitamin D have been
associated with symptoms of depression and mood swings.

Impaired Immune Function: Vitamin D is involved in modulating immune responses. Inadequate vitamin D
levels can compromise immune function and increase the susceptibility to infections, such as respiratory tract
infections and autoimmune conditions.

Bone Deformities (in Children): In growing children, vitamin D deficiency can lead to rickets, a condition
characterized by weakened and deformed bones. This can cause growth delays, bowing of the legs, and skeletal
abnormalities.

Increased Risk Factors: Certain individuals are at a higher risk of vitamin D deficiency. These include people with
limited sun exposure, individuals with dark skin (as melanin reduces the skin's ability to produce vitamin D),
older adults, obese individuals, those with malabsorption issues (e.g., Crohn's disease, celiac disease), and
individuals with limited dietary intake of vitamin D-rich foods

Q. 85 - An alcoholic patient presented to opd with ataxia confusion & visual changes. Identify the cause of the
disease.

A. Thiamine B. Niacin
C. Biotin D. Vit a

61
Correct Answer - A
Explanation
Chronic alcoholism can lead to thiamine (vitamin B1) deficiency. Thiamine is an essential nutrient required for
proper functioning of the nervous system, metabolism, and energy production. Alcohol can interfere with
thiamine absorption, storage, and utilization in the body, leading to a deficiency. Here's how chronic alcoholism
can contribute to thiamine deficiency:

Poor Dietary Intake: Alcoholics often have poor diets that lack essential nutrients, including thiamine. Alcohol
itself provides calories but has little nutritional value, so it can displace nutrient-rich foods from the diet,
leading to deficiencies.

Impaired Absorption: Alcohol can damage the cells lining the stomach and intestines, which are responsible for
absorbing thiamine and other nutrients. This can hinder the absorption of thiamine from the diet, even if it is
consumed adequately.

Decreased Storage and Utilization: Chronic alcohol consumption can deplete the body's thiamine stores and
impair its utilization. Alcohol interferes with the enzymes involved in thiamine metabolism, reducing the
conversion of thiamine into its active form (thiamine pyrophosphate). As a result, the body's ability to use
thiamine effectively is compromised.

Increased Thiamine Excretion: Alcohol can increase thiamine excretion through urine, further depleting
thiamine levels in the body.

Thiamine deficiency resulting from chronic alcoholism can lead to a condition known as Wernicke-Korsakoff
syndrome (WKS). WKS is a neurological disorder that consists of two distinct stages:

Wernicke's Encephalopathy: This acute stage is characterized by confusion, ataxia (loss of muscle coordination),
ophthalmoplegia (paralysis of eye muscles), and nystagmus (involuntary eye movements). It can also involve
symptoms such as memory loss, disorientation, and impaired cognitive function.

Korsakoff's Syndrome: This chronic stage involves severe memory impairment, confabulation (fabrication of
false memories), and difficulty learning new information. Individuals with Korsakoff's syndrome may have
significant gaps in their memory and struggle with daily functioning.

62
Thiamine replacement therapy is crucial for individuals with alcohol-related thiamine deficiency. Intravenous or
oral thiamine supplements are typically administered to replenish thiamine levels and prevent or treat
Wernicke-Korsakoff syndrome. If left untreated, Wernicke-Korsakoff syndrome can lead to permanent brain
damage and disability.

Q. 86 - A pt. Came with TB Chest symptom with negative sputum sample for TB. chest x-ray findings are
suggestive of TB. what is next step of management?

A. Repeat sputum samples B. Start TB treatment


C. CBNAAT D. Home remedy

Correct Answer - A
Explanation
People can have TB even if the sputum culture results come back as negative. Active tuberculosis is an infection
with Mycobacterium tuberculosis which can be transmitted to other people. Usually this is an infection of the
lungs (pulmonary tuberculosis), it can be spread through droplets while coughing and sneezing.

If X-ray are suggesting towards the diagnosis of TB & suptum is negative high chances are there that it could be
false negative in such condition its better to repeat the test and then go ahead with any further investigation.
Next Best step is to go for CBNAAT.

Q. 87 - A clean cut wound in a male with TT received 10 years ago. What should be the management of
wound.

A. Full course tetanus vaccine to be given B. Full dose TT with TIG


C. Single dose TT D. No treatment required

63
Correct Answer - C
Explanation

Q. 88 - A patient has varicella. The management for remaining contacts would be

A. Isolate for 6 days + acyclovir + vzig for 72 hours B. VZIG only


C. Isolate for 14 days + vzig within 48 hours D. Isolate for 6 days

Correct Answer - C
Explanation
VZV-seronegative immunocompromised patients with a defined close exposure to either chickenpox or herpes
zoster should receive VZIG to provide passive immunity. In most cases, VZIG administration will not prevent
infection in the susceptible host, but it will significantly reduce the severity of the resultant illness.
It should be administered within 48-72 hours along with the isolation.

Q. 89 - A baby born with prematurity, has low birth weight, decreased hearing, PDA, bilateral cataract, What
is the most likely cause?

A. Rubella B. CMV
C. Toxoplasmosis D. HSV

64
Correct Answer - A
Explanation
Based on the provided information, the most likely cause of these multiple conditions in a premature baby with
low birth weight, decreased hearing, patent ductus arteriosus (PDA), and bilateral cataract is Congenital Rubella
Syndrome (CRS). CRS occurs when a pregnant woman contracts the rubella (German measles) virus during the
early stages of pregnancy, which can lead to severe developmental abnormalities in the fetus.

Here's how each condition is associated with CRS:

Prematurity and Low Birth Weight: Infections during pregnancy, such as rubella, can increase the risk of
premature birth and result in a low birth weight.

Decreased Hearing: Sensorineural hearing loss is a common feature of CRS. The rubella virus can affect the
development of the auditory system, leading to varying degrees of hearing impairment.

Patent Ductus Arteriosus (PDA): PDA is a heart condition in which the ductus arteriosus, a blood vessel that
bypasses the lungs during fetal development, remains open after birth. PDA is more common in infants with
CRS, as the rubella virus can cause cardiac abnormalities.

Bilateral Cataract: The rubella virus can affect the development of the eyes, leading to congenital cataracts.
Bilateral cataracts refer to the presence of cataracts in both eyes

Q. 90 - A poor farmer with a history of successive crop failures develops progressive spastic paraparesis,
upper motor neuron signs & gait instability. Most likely toxin responsible.

A. BOAA B. Aflatoxin
C. Strychnine D. Ergotoxine

Correct Answer - A
Explanation
Lathyrism is a chronic toxic nutritional neurological disease caused by long-term (or subacute) ingestion of flour
made from the drought-resistant chickling pea (Lathyrus sativus). It is an important example of a disease in
which a natural excitotoxin causes selective UMN impairment. The responsible neurotoxin is
β-N-oxalylamino-l-alanine (BOAA), an α-amino-3-hydroxyl-5-methyl-4-isoxazolepropionic acid (AMPA)
glutamate
65
receptor agonist. Ingestion of this neurotoxin results in increased intracellular levels of reactive oxygen species
and subsequent impairment of the mitochondrial oxidative phosphorylation chain.

The consumption of large quantities of Lathyrus seeds containing high concentrations of the neurotoxic
glutamate analogue β-oxalyl-L-α,β-diaminopropionic acid (ODAP, also known as β-N-oxalyl-amino-L-alanine, or
BOAA) causes paralysis, characterized by lack of strength in or inability to move the lower limbs, and may
involve pyramidal tracts, producing signs of upper motor neuron damage.

The toxin may also cause aortic aneurysm. A unique symptom of lathyrism is the atrophy of gluteal (buttocks)
muscles. ODAP is a poison of the mitochondria, leading to excess cell death, especially in motor neurons.
Children can additionally develop bone deformity and reduced brain development.

Q. 91 - Which vitamin is used to treat neurolathyrism

A. Vit C B. Vit A
C. Vit B D. Vit E

Correct Answer - A
Explanation
Vit C is used to treat neurolathyrism.
Multiple studies have been conducted on guinea pigs in support of this statement.

one such study was conducted by W.B Bunham et. al which showed that protection afforded by dietary vitamin
C from the toxic effects of intraperitoneal injection of an aqueous solution of an air-dried ethanol extract of
Lathyrus sativus peas is demonstrated in guinea pigs fed an ascorbate-fortified diet in contrast to those fed a
diet devoid of vitamin C.

Weakness and flaccid paraparesis occurred in most of the ascorbate-depleted animals, whereas all guinea pigs
on the ascorbate-supplemented diet remained active and appeared well following injection of the extract.
These results extend previous work on the role of dietary ascorbate in the protection of guinea pigs from
neurolathyrism.

66
In particular, spastic paralysis in some ascorbate-depleted guinea pigs is reported for the first time. Since guinea
pigs, like all primates, including humans, do not synthesize vitamin C, their use as experimental models can
partially simulate human neurolathyrism.

Q. 92 - A person with low HTN in childhood tend to have low BP in adult

A. Tracking phenomenon B. Rule of Halve


C. WHO Steps approach D. None of the above

Correct Answer - A
Explanation
individuals with low blood pressure (hypotension) during childhood tend to have low blood pressure in
adulthood, can be considered a tracking phenomenon. Tracking refers to the persistence or stability of a certain
characteristic or condition over time.

In the case of blood pressure, studies have suggested that there is a degree of tracking of blood pressure levels
from childhood to adulthood. Individuals who have low blood pressure during childhood are more likely to
maintain lower blood pressure levels as they grow older.

Several factors contribute to this tracking phenomenon:

Genetic Factors: Genetic predisposition plays a role in determining an individual's blood pressure levels. Family
history of low blood pressure may increase the likelihood of having low blood pressure throughout life.

Lifestyle Factors: Lifestyle habits developed during childhood can impact blood pressure in adulthood.
Individuals who maintain a healthy lifestyle, including regular physical activity, a balanced diet, and avoidance of
smoking and excessive alcohol consumption, are more likely to have lower blood pressure throughout their
lives.

Body Weight and Body Mass Index (BMI): Maintaining a healthy body weight and BMI can contribute to lower
blood pressure levels. Childhood obesity is associated with an increased risk of developing high blood pressure
67
in adulthood. Conversely, maintaining a healthy weight in childhood may lead to lower blood pressure in
adulthood.

Cardiovascular Health: Good cardiovascular health in childhood, including healthy heart function and blood
vessel elasticity, may contribute to lower blood pressure in adulthood.

Q. 93 - Which disease is caused by zinc deficiency

A. Acrodermatitis enteropathica B. Psoriasis


C. Seborrheic dermatitis D. Glucagonoma syndrome.

Correct Answer - A
Explanation
Acrodermatitis enteropathica is a rare inherited disorder that affects the body's ability to absorb zinc from the
diet. It is characterized by a deficiency of zinc, an essential mineral necessary for various physiological
processes.

Characteristics features of acrodermatitis enteropathica:

Dermatological Symptoms: The condition primarily manifests with skin abnormalities. The most characteristic
dermatological feature is a rash that typically affects the face, extremities (hands and feet), and perineal area.
The rash can present as red, inflamed, and scaly patches, often resembling eczema or psoriasis.

Diarrhea: Individuals with acrodermatitis enteropathica commonly experience chronic diarrhea. Zinc deficiency
affects the integrity of the intestinal lining and impairs the absorption of other nutrients as well.

Failure to Thrive: Infants and young children with acrodermatitis enteropathica often fail to thrive, meaning
they have difficulties gaining weight and growing at a normal rate. This can be attributed to the compromised
nutrient absorption caused by zinc deficiency.

68
Poor Appetite: Reduced appetite and decreased food intake are common in individuals with acrodermatitis
enteropathica. This can further contribute to malnutrition and impaired growth.

Delayed Wound Healing: Zinc is essential for wound healing and tissue repair. In acrodermatitis enteropathica,
the deficiency of zinc can lead to delayed wound healing and an increased susceptibility to skin infections.

Behavioral and Neurological Changes: In severe cases or when the condition is left untreated, acrodermatitis
enteropathica can lead to neurological symptoms such as irritability, emotional instability, and developmental
delays.

Acrodermatitis enteropathica is a genetic disorder caused by mutations in the SLC39A4 gene, which encodes a
protein involved in zinc transport. It is typically inherited in an autosomal recessive manner

Q. 94 - Why do you get bleeding manifestation. Which Vit Deficiency causes this?

A. Vit A B. Vit C
C. Vit D D. Vit K

Correct Answer - B
Explanation
Vitamin C deficiency, also known as scurvy, is a disease primarily associated with socioeconomic status and
access to food.

Symptoms include fatigue, depression, and connective tissue defects (eg, gingivitis, petechiae, rash, internal
bleeding, impaired wound healing). In infants and children, bone growth may be impaired.

Vitamin C deficiency can affect blood vessels and potentially contribute to nosebleeds:

Collagen Formation: Vitamin C is essential for the synthesis of collagen, a protein that provides strength and
structure to blood vessels. Inadequate vitamin C levels can impair collagen production, weakening the integrity
of blood vessel walls.

Weakened Blood Vessel Walls: Without sufficient collagen, blood vessels become fragile and prone to damage.
Weakened blood vessel walls can make them more susceptible to rupture, which may lead to nosebleeds.

69
Capillary Fragility: Vitamin C deficiency can also affect capillaries, the smallest blood vessels in the body.
Capillaries become more fragile without proper collagen support, increasing the risk of bleeding and potentially
contributing to nosebleeds.

Skin
Q. 95 - A 50 year old male presented with a h/o HIV & painful lesion on his palate. Identify the given disease?

A. Kaposi sarcoma B. Squamous cell carcinoma


C. Basal cell carcinoma D. HCC

Correct Answer - A
Explanation
Kaposi's sarcoma (KS) is an uncommon, multifocal and angioproliferative lesion, which demonstrates a poor
prognosis.
HIV viral load, CD4+ count and the CD4+/CD8+ ratio are associated with oral KS development.

Kaposi’s sarcoma (KS) is a form of cancer in which tumors with tiny blood vessels grow below the surface of
skin, mouth, nose, eyes, and anus.
It can spread to lungs, liver, stomach, intestines, and lymph nodes, the glands that help body fight infection.

Types of Kaposi’s Sarcoma

There are four types:

● Epidemic or AIDS-associated. This is the most common kind in the U.S. It affects people
who have HIV. It’s known as an AIDS-defining illness because it’s on the CDC’s list of
conditions that mean someone’s HIV infection has become AIDS.
● Classic. This type affects older men of Mediterranean, Middle Eastern, or Eastern
European descent.
● Endemic. Children and young people from Africa get this kind of KS.
● Immunosuppressive. This kind affects people who have had organ transplants and take
drugs that slow down their immune system.

70
Q. 96 - Obese female presenting with acne on face. Even after taking medication her acne are not getting
better. Identify the cause?

A. Hypergonadism B. Change in her diet


C. PCOS D. Uses of lotions & cream

Correct Answer - A
Explanation
Hypergonadism is a condition where there is a hyperfunction of the gonads. It can manifest as precocious
puberty, and is caused by abnormally high levels of testosterone or estrogen, crucial hormones for sexual
development

Hypergonadism that develops prior to puberty can lead to precocious puberty. Precocious puberty is the early
and rapid onset of changes related to sexual maturity. Hypergonadism is one of several possible causes of
precocious puberty.

Risk factors for the development of acne, other than genetics, include hormones, infections, diet, and stress.

In boys and girls, hypergonadism can bring on:

● early growth spurts


● mood swings
● acne
● a lower voice

Q. 97 - Which chemical causes following allergic reaction

A. Pollen B. PPD
C. Balsum of peru D. Hexa
71
Correct Answer - B
Explanation
Hair coloring products contain many ingredients that can irritate the skin and cause allergic reactions. Most
cases of allergic contact dermatitis stemming from exposure to hair dye are caused by an ingredient called
paraphenylenediamine (PPD).

PPD is a chemical that’s also found in temporary tattoo ink, printer ink, and gasoline. In boxed hair dye, PPD
usually comes in its own bottle, accompanied by an oxidizer.

When both are mixed together, PPD becomes partially oxidized. This is when it’s likely to cause allergic reactions
in people who are sensitive to it.

Q. 98 - A patient with diarrhea, dementia, and dermatitis. Which nutritional deficiency causes this?

A. Niacin B. Biotin
C. Vit B9 D. Vit A
Correct Answer - A
Explanation
Symptoms of a slight niacin deficiency can be so mild at first. But if this deficiency continues, it can lead to a
condition called pellagra.

Pellagra has some key characteristics known as "the four Ds," which generally appear in this order:

1. Diarrhea
2. Dermatitis (skin irritation)
3. Dementia
4. Death

Signs and symptoms of pellagra can include:

● Nausea, vomiting
● Diarrhea, constipation
● Bright red tongue
● Rash that resembles sunburn and may darken in sunlight

72
● Blisters, skin sloughing (peeling, shedding)
● Headaches
● Insomnia, fatigue
● Depression, hallucinations, paranoia
● Suicidal behavior
● Aggressiveness
● Memory loss, apathy
● Loss of appetite

Without treatment, pellagra can lead to death within a few years.

Q. 99 - What is the long term complication of this condition

A. Malignant melanoma B. Cataract


C. Loss of vision D. Glucoma

Correct Answer - A
Explanation

The congenital melanocytic nevus appears as a circumscribed, light brown to black patch or plaque, potentially
very heterogeneous in consistency, covering any size surface area and any part of the body.

As compared with a melanocytic nevus, congenital melanocytic nevi are usually larger in diameter and may
have excess terminal hair, a condition called hypertrichosis. If over 40 cm (16 in) projected adult diameter with
hypertrichosis, it is sometimes called giant hairy nevus; more usually these largest forms are known as large or
giant congenital melanocytic nevus.

A giant congenital melanocytic nevus (GCMN) is found in 0.1% of live-born infants.


If present, the lesion has a chance of about 6% to develop into malignant melanoma.

Both children and adults can be affected by malignant melanoma arising in a giant congenital nevus.

73
Up to 95% of GCMNs harbor NRAS mutations, and mutations in the BRAF, MC1R, TP53, and GNAQ genes have
also been described.

Q. 100 - What is the cause of painless ulcer associated with painless lymph node

A. Granuloma Inguinale B. Chancroid


C. Syphilis D. Neisseria gonorrhea

Correct Answer - C
Explanation
A chancre is a painless genital ulcer most commonly formed during the primary stage of syphilis. This infectious
lesion forms around 21 days after the initial exposure to Treponema pallidum, the gram-negative spirochaete
bacterium causing syphilis, but can range from 10 to 90 days. Without treatment it may persist for two to six
weeks before healing. Chancres transmit syphilis through direct physical contact. These ulcers usually form on
or around the anus, mouth, penis and vulva.

The chancre appears as an indurated, punched-out, painless ulcer. It is usually accompanied by painless
lymphadenopathy. This resolves without scarring after 1–5 weeks. The secondary cutaneous lesions (syphilids)
are highly infectious and may mimic virtually any skin disorder.

Q. 101 -Wife of a truck driver with multiple sexual partners presented with vaginal discharge & on
examinations cervical erosions were present. Which STD Kit we will give in this patient.

A. Green B. Grey
C. Red D. Yellow

Correct Answer - A
Explanation

Kit No. Syndrome Colour Contents

Kit 1 Urethral Grey Tab.


discharge Azithromy
(UD), cin 1 g (1)
Cervical and Tab.
discharge Cefixime
(CD) , 400 mg (1)

74
Ano-rectal
discharge
(ARD)
Painful
scrotal
swelling
(PSS)
Presumpti
ve
treatment
(PT)

Kit 2 Vaginal Green Tab.


discharge Secnidazol
(VD) e 2 g (1)
and Tab.
Fluconazol
e 150 mg
(1)

Kit 3 Genital White Inj.


Ulcer Benzathin
Disease- e penicillin
Non 2.4 MU (1)
herpetic and Tab.
(GUD-NH) Azithromy
cin 1 g (1)
and
Disposable
syringe 10
ml with 21
gauge
needle (1)
and Sterile
water 10
ml (1)

Kit 4 Genital Blue Tab.


Ulcer Doxycyclin
Disease- e 100 mg
Non-Herp (30) and
etic Tab.
(GUD-NH) Azithromy
–for cin 1 g (1)
patients
allergic to
penicillin.

Kit 5 Genital Red Tab.


Ulcer Acyclovir
Disease- 400 mg
Herpetic

75
(GUD-H)

Kit 6 Lower Yellow Tab.


abdominal Cefixime
pain 400 mg
(LAP/PID) and
Tab.Metro
nidazole
400 mg
and Cap.
Doxycyclin
e 100 mg

Kit 7 Inguinal Black Tab.


bubo (IB) Doxycyclin
e 100 mg
and Tab.
Azithromy
cin 1 g

76
Psychiatry

Q. 102 - A 25 year old female patient is brought with symptoms of anxiety, palpitations, sweating,
breathlessness, chest pain, feeling she might have a heart attack/ feeling of impending doom. Patient has 5-6
episodes in a month, 30 min each since 6 months. What is the diagnosis?

A. Depression B. Panic disorder


C. GAD D. Agoraphobia

Correct Answer - A
Explanation
People with panic disorder have frequent and unexpected panic attacks. These attacks are characterized by a
sudden wave of fear or discomfort or a sense of losing control even when there is no clear danger or trigger.

Panic attacks often include physical symptoms that might feel like a heart attack, such as trembling, tingling, or
rapid heart rate. Panic attacks can occur at any time. Many people with panic disorder worry about the
possibility of having another attack and may significantly change their life to avoid having another attack.

Panic disorder often begins in the late teens or early adulthood. Women are more likely than men to develop
panic disorder.

Q. 103 - A male patient comes with a history of marital conflicts and complains of early ejaculation. Which of
the following non pharmacological method may be useful in this condition?

A. Squeeze technique B. CBT


C. Exposure and responsive prevention D. Sensate focus technique

Correct Answer - A
Explanation

77
In the squeeze technique, the penis is also stimulated until just before orgasm. Then you put a little bit of
pressure on the head of the penis to decrease the level of arousal. This can be done by putting your index finger
on the back side of the penis (where the head joins the shaft) and placing your thumb on the other side of the
penis, and then gently squeezing. After that you wait for about 30 seconds and then repeat the steps several
times in a row.

Q. 103 - A patient with a diagnosis of schizophrenia, not responding to haloperidol and thioridazine. Started
on another medication. Drug A with side effects of sialorrhea, dyslipidemia, hyperglycemia. What drug
started?

A. Clozapine B. Risperidone
C. Aripiprazole D. Ziprasidone

Correct Answer - A
Explanation
The new medication described has side effects of sialorrhea (excessive salivation), dyslipidemia (abnormal
levels of lipids in the blood), and hyperglycemia (high blood sugar). One medication commonly associated with
these side effects is clozapine.

Clozapine is an atypical antipsychotic medication often prescribed for treatment-resistant schizophrenia. It is


known to have a higher risk of metabolic side effects compared to other antipsychotics. Sialorrhea
(hypersalivation) is a common side effect of clozapine. Dyslipidemia and hyperglycemia are also potential
metabolic side effects associated with clozapine use.

Q. 104 - A 75 year old female has difficulty in doing activities. She has resting tremors, rigidity, visual
hallucination and looking into the sky. Lewy body was seen. Diagnosis is?

A. Parkinson B. Alzheimer disease


C. Prion disease D. Huntington's disease

Correct Answer - C
Explanation
Prion diseases, a group of disorders caused by abnormally shaped proteins called prions, occur in sporadic
(Jakob-Creutzfeldt disease), genetic (genetic Jakob-Creutzfeldt disease, Gerstmann-Sträussler-Scheinker

78
syndrome, and fatal familial insomnia), and acquired (kuru, variant Jakob-Creutzfeldt disease, and iatrogenic
Jakob-Creutzfeldt disease) forms.

Dementia with Lewy bodies (DLB) and multiple system atrophy (MSA) are caused by α-synuclein prions

Radiology

Q. 105 - A 31-year-old female delivered a 1 kg baby at 31 weeks gestation. The child has hepatosplenomegaly
& jaundice. On CT periventricular calcification is seen. What is the most likely diagnosis?

A. Toxoplasmosis B. Herpes Simplex


C. CMV D. Parvo B 19

Correct Answer - C
Explanation
The most likely diagnosis based on the information provided is Congenital Cytomegalovirus (CMV) infection.

CMV is a common viral infection that can be transmitted from a mother to her baby during pregnancy. Infants
who are infected with CMV may have hepatosplenomegaly (enlarged liver and spleen), jaundice, and
periventricular calcifications on CT. Premature birth is also a risk factor for CMV infection.

**Note that other congenital infections can also cause similar symptoms, so further diagnostic testing may be
necessary to confirm the diagnosis.

Q. 106 - A 32-year-old presents with an altered sensorium. CT reveals bi-temporal hemorrhage. What is the
diagnosis?

A. TB B. Herpes Simplex
C. Brain cancer D. Stroke

Correct Answer - B
Explanation

79
A bi-temporal hemorrhage on a CT scan can be indicative of various conditions that cause bleeding in the brain.
Herpes simplex virus (HSV) infection is not a common cause of bitemporal hemorrhage, but it can cause
encephalitis (inflammation of the brain) which can lead to altered sensorium.

Herpes simplex encephalitis (HSE) is a rare but serious condition that can occur as a result of HSV infection. It
can cause inflammation of the temporal lobes of the brain, which may present as a bi-temporal hemorrhage on

imaging studies. The symptoms of HSE can include altered sensorium, fever, headache, seizures, and
neurological deficits such as aphasia (difficulty speaking) or hemiparesis (weakness on one side of the body).

Other \ causes of bitemporal hemorrhage, such as head trauma, intracranial bleeding due to hypertension,
cerebral aneurysm, and other infectious or inflammatory conditions affecting the brain.

Therefore, it is important to perform additional diagnostic tests, such as blood tests, CSF analysis, and imaging
studies, to determine the underlying cause of the hemorrhage and the appropriate treatment. A thorough
physical exam and a detailed medical history can also help in the diagnosis of the underlying condition.

Q. 107 - A 22-year-old female presented with swelling in her forearm. Radiograph is given below. What is the
diagnosis?

A. Giant cell tumor B. Chondroblastoma


C. Osteochondroma D. Aneurysmal Bone cyst

Correct Answer - A
Explanation
A "soap bubble appearance" on a radiograph is a term used to describe the appearance of certain bone lesions,
which can be seen in a variety of conditions. One of the most common bone tumors that can present with a
soap bubble appearance is a giant cell tumor (GCT) of bone.

80
Giant cell tumors of bone are usually benign but locally aggressive tumors that can occur in any bone, but most
commonly affect the long bones of the extremities, such as the forearm. The tumor is composed of
multinucleated giant cells and stromal cells, and can cause extensive bone destruction.

On radiographs, GCTs typically present as well-defined, osteolytic lesions with a "soap bubble" appearance,
which refers to the presence of multiple fluid-filled cavities within the tumor.

Treatment for GCTs of bone usually involves surgical resection, although non-surgical options such as
denosumab may be considered for unresectable or metastatic tumors.

Q. 108 - A 35-year-old patient with morning stiffness & reduced chest expansion. Also the patient had a
complaint of red eyes. What is the diagnosis on the basis of history & radiographs?

A. Healed TB B. Ankylosing spondylitis


C. Pagets disease D. Osteoporosis

Correct Answer - B
Explanation
The combination of morning stiffness, reduced chest expansion, and red eye in a 35-year-old patient is highly
suggestive of ankylosing spondylitis (AS), a chronic inflammatory disorder that primarily affects the spine and
sacroiliac joints.

Morning stiffness is a hallmark symptom of AS, as it tends to be worse in the morning or after periods of
inactivity. Reduced chest expansion is also a common feature of AS, as inflammation in the spine and rib cage
can lead to limited chest expansion and difficulty breathing. The red eye is another common symptom of AS,
known as uveitis, which is inflammation of the eye's uvea (the middle layer of the eye).

Radiographs can help confirm the diagnosis of AS by showing characteristic changes in the spine and sacroiliac
joints, such as erosions, sclerosis, and fusion of the joints.
81
Early diagnosis and treatment of AS is important to prevent or delay progression of the disease and preserve
mobility. Treatment options include nonsteroidal anti-inflammatory drugs (NSAIDs), disease-modifying
antirheumatic drugs (DMARDs), and biologic agents such as TNF inhibitors. Physical therapy and exercise are
also important components of treatment to maintain mobility and prevent joint stiffness.

Q. 109 - 45-year-old female who has to undergo radiotherapy for treatment of Ca cervix stage 2b. Which of
the following statements is true regarding ionizing radiation?

A. The normal cell & cancer cell will be equally sensitive


B. GI mucosa is more resistant part
C. The radiation at a point is inversely proportional to the square of the distance from the source
D. Small blood vessels are resistant to the effect of radiation

Correct Answer - C
Explanation
The statement "the radiation at a point is inversely proportional to the square of the distance from the source"
is a simplified version of the inverse square law, which describes how the intensity of radiation or light
decreases as the distance from the source increases. This law applies to many types of radiation, including
ionizing radiation used in medical treatments such as radiotherapy for cancer.

In the case of radiotherapy for cervical cancer, the radiation source is usually a linear accelerator that produces
high-energy X-rays or electrons that are directed at the tumor site to kill cancer cells.

According to the inverse square law, the intensity of radiation decreases by a factor of the square of the
distance from the source. This means that if the distance between the source and a particular point is doubled,
the intensity of radiation at that point will be reduced to one-fourth of the original value. Similarly, if the
distance is tripled, the intensity will be reduced to one-ninth of the original value, and so on.

Q. 110 - A 3-year-old child presented with fever & biphasic stridor. There is a history of prior URI. There is no
dysphagia. What is the diagnosis on the basis of radiographs given below

82
A. Acute laryngotracheobronchitis B. Acute epiglottitis
C. Laryngomalacia D. Foreign body glottis

Correct Answer - A
Explanation
Acute laryngotracheobronchitis, also known as croup, is a viral infection that affects the upper respiratory tract.
It is a common childhood illness that typically occurs in children between 6 months and 3 years of age.

Croup is characterized by inflammation of the larynx (voice box), trachea (windpipe), and bronchi (large airways
leading to the lungs). This inflammation can cause symptoms such as a barking cough, hoarseness, stridor (a
high-pitched breathing sound), and difficulty breathing.

Treatment for croup may include the use of humidifiers, nebulized epinephrine, and oral or inhaled steroids to
reduce inflammation and improve breathing. Antibiotics are not usually recommended as croup is usually
caused by a viral infection. If a bacterial infection is suspected, however, antibiotics may be prescribed.

Q. 111 - A 36-year-old patient with cough & fever for the last 12 days. There are crepts on the right side.
X-ray is done. What is the diagnosis?

A. Apical segment of lower lobe consolidation


B. Posterior segment of lower lobe consolidation
C. Middle lobe consolidation
D. Loculated pleural effusion

Correct Answer - C
83
Explanation
If the patient has been diagnosed with middle lobe consolidation based on further imaging studies such as a
chest X-ray or computed tomography (CT) scan, this would suggest that the pneumonia is localized to the
middle lobe of the right lung.

Middle lobe consolidation is a type of pneumonia that affects the middle lobe of the right lung. It can be caused
by a variety of infectious agents such as bacteria, viruses, and fungi. The symptoms of middle lobe pneumonia
are similar to those of pneumonia in general and may include cough, fever, chills, chest pain, and shortness of
breath.

The treatment of middle lobe pneumonia would typically involve antibiotics to target the specific infectious
agent causing the pneumonia.

Q. 112 - A CT scan of a 50-year-old patient is given who is presented with abdominal pain. Which of the
following statements is correct?

A. Surgery is mandatory B. Etiology is echinococcus spp.


C. Angioembolization is indicated D. FNAC is diagnostic

Correct Answer - B
Explanation
If the CT scan of a 50-year-old patient with abdominal pain reveals the presence of echinococcus spp., this
would suggest that the patient has an echinococcal cyst or hydatid cyst in the abdomen.

Echinococcus spp. is a type of parasitic tapeworm that can infect humans. The infection occurs when a person
ingests the eggs of the tapeworm, which are typically found in the feces of infected dogs or other animals. The
eggs can then hatch in the person's intestine and the larvae can migrate to other parts of the body, such as the
liver, lungs, or abdominal cavity, where they form cysts.
84
The symptoms of echinococcal cysts include abdominal pain, nausea, vomiting, and fever.
Treatment for echinococcal cysts typically involves surgical removal of the cyst, along with the use of
antiparasitic medication to kill any remaining tapeworm larvae.

Q. 113 - A 30-year-old female presented with non axial proptosis of a left eye for the last 3 years. There is a
history of trauma to the eye 10 years back. The imaging reveals a homogenous lesion in the left frontal sinus
as given below. What is the diagnosis?

A. Frontal mucocele B. Frontal meningioma


C. Orbital pseudotumor D. Juvenile nasopharyngeal angiofibroma

Correct Answer - A
Explanation
Based on the provided information, frontal mucocele could be a possible diagnosis for the homogenous lesion
seen in the left frontal sinus of the 30-year-old female with non axial proptosis of the left eye.

A mucocele is a benign, cystic lesion that develops from the accumulation of mucus within a paranasal sinus.
Frontal mucoceles specifically arise from the frontal sinus, which is located above the eyes in the forehead. They
typically develop as a result of obstruction of the drainage pathway from the frontal sinus into the nasal cavity,
leading to the accumulation of mucus and expansion of the sinus.

Symptoms of frontal mucoceles can include headache, facial pain or pressure, nasal congestion, and visual
disturbances such as proptosis or double vision. Treatment for frontal mucoceles typically involves surgical
removal of the cyst and repair of the sinus drainage pathway to prevent recurrence.

Q. 114 - A radiograph of a 10 Year old boy is given below who has had knee pain for the last 3 months. There
is no definitive history of trauma. What is the correct statement regarding diagnosis & management
85
A. Fracture of upper pole
B. Avulsion fracture of patella- tension band wiring
C. Bipartite patella- Do radiograph of the other knee
D. Avulsion fracture of patella- Interfragmentary screw fixation

Correct Answer - C
Explanation
The Age group as well as there is no history of any trauma gives a diagnosis in favor of bipartite patella. Since
it is bilateral in 50% of the cases, so an xray of the other knee can be done.

A bipartite patella (two-part patella) is a patella with an unfused accessory ossification center, typically at the
superolateral aspect.

Epidemiology
The superolateral accessory ossification center of the patella is usually present by 12 years of age and may
persist into adult life. Bipartite patella occurs in approximately 2% of the population, and occurs bilaterally in
about 43% of cases. It is 9 times more common in males than in females

Clinical presentation
A bipartite patella is usually discovered incidentally in asymptomatic individuals. Only 2% of patients with
bipartite patella experience symptoms. It may cause anterior knee pain, especially after trauma, sports injury,
or overuse

Treatment and prognosis


In the majority of cases, symptomatic bipartite patella improves without surgery. Surgical excision of the
small fragment is recommended if conservative management fails and has been reported to give good results

86
Q. 115 - A construction worker developed a persistent cough along with shortness of breath. Also he
developed mesothelioma. What is the diagnosis?

A. Asbestosis B. Lung cancer


C. COPD D. Chronic Bronchitis

Correct Answer - A
Explanation
Based on the provided information, asbestosis could be a possible diagnosis for the construction worker with
persistent cough, shortness of breath, and mesothelioma.

Asbestosis is a chronic lung disease that results from prolonged exposure to asbestos fibers, which are
commonly found in construction materials such as insulation, roofing, and flooring. The fibers can become
trapped in the lungs and cause inflammation and scarring over time, leading to difficulty breathing, persistent
cough, and chest pain.

Exposure to asbestos is also associated with an increased risk of developing mesothelioma, a type of cancer
that affects the lining of the lungs, chest cavity, or abdomen. Symptoms of mesothelioma can include shortness
of breath, chest pain, cough, and weight loss.

Q. 116 - A patient with bone deformity as given in the image & also with skin lesions. What could be the
possible diagnosis

A. Fibrous Dysplasia B. GCT


C. Enchondroma D. Pagets Disease

Correct Answer - A
87
Explanation
Based on the provided information and image of the bone deformity and skin lesions, fibrous dysplasia could be
a possible diagnosis for the patient.

Fibrous dysplasia is a rare bone disorder that causes abnormal growth and development of bone tissue, leading
to bone deformities, fractures, and pain. It can affect one or multiple bones in the body, and may be associated
with other conditions such as skin lesions and endocrine abnormalities.

The characteristic appearance of fibrous dysplasia on radiographs is a "ground-glass" or "cotton wool"


appearance, which may be seen in the image provided. In some cases, there may also be areas of calcification
or cystic changes within the affected bone.

Skin lesions may also be present in patients with fibrous dysplasia, such as café-au-lait spots or patches of
hyperpigmentation. These skin findings are often seen in patients with McCune-Albright syndrome, a variant of
fibrous dysplasia that is associated with endocrine abnormalities such as precocious puberty and
hyperthyroidism.

Q. 117 - Identify the fracture in the given radiograph which causes the inability of dorsiflexion

A. Femur fracture B. Tibia fracture


C. Fibula fracture D. B & c

Correct Answer - C
Explanation
A fibula fracture typically does not cause the inability of dorsiflexion (lifting the foot upwards towards the shin),
as it is not directly involved in this movement. However, the fibula is an important bone in the lower leg that
provides structural support to the ankle joint and helps to stabilize the foot and ankle during movement.

If the fibula fracture is severe or involves other bones or soft tissues in the ankle or foot, it may result in
swelling, pain, and limited range of motion, including limited dorsiflexion. The common peroneal nerve is the
88
nerve that runs through the neck of the fibula, which is a bony groove located at the top of the fibula bone in
the lower leg.

The common peroneal nerve is a branch of the sciatic nerve and provides motor and sensory innervation to the
muscles and skin of the lower leg and foot. It is particularly vulnerable to injury as it winds around the neck of
the fibula, and trauma or compression in this area can cause damage to the nerve.

Q. 118 - A pedo patient reported swelling on the head since birth (2 months). What is the diagnosis

A. Cephalhematoma B. Caput succedaneum


C. Soft tissue swelling D. None of the above
Correct Answer - A
Explanation
If a pediatric patient presents with a swelling on the head that has been present since birth, cephalhematoma is
one possible diagnosis that should be considered.

Cephalhematoma is a type of birth injury that occurs when bleeding occurs between the skull and the
membrane that covers it, called the periosteum. This can cause a raised, soft, and fluctuant area on the head
that does not cross suture lines. The swelling is typically limited to one side of the head, and the size of the
swelling may increase over the first few days after birth before gradually resolving over several weeks to
months.

Calcified cephalohematoma is a rare complication that can occur in some cases of cephalohematoma. A
cephalohematoma is a collection of blood that occurs between the skull and the periosteum, the membrane
that covers the skull bone. This condition usually occurs due to birth trauma, such as the use of forceps during
delivery.

89
When the cephalohematoma heals, the body usually reabsorbs the blood. However, in some cases, the blood
may calcify and form a hard, bony lump on the skull. This can occur because the body tries to replace the lost
blood with calcium, leading to the formation of bone tissue.

A calcified cephalohematoma is usually harmless and does not cause any symptoms. However, in rare cases, it
may cause cosmetic deformities or require surgical removal if it is large or causes pressure on the brain.

If a calcified cephalohematoma is suspected, further evaluation with imaging studies such as X-rays or CT scans
may be necessary to confirm the diagnosis and assess the extent of the calcification. Treatment will depend on
the size and location of the calcified area and the potential for complications. In most cases, however, no
treatment is necessary, and the calcified area can be monitored for any changes over time.

90
Anaesthesia

Q. 119 - When to put patient in given position?

Unconscious patient and both pulse & breathing absent


Unconscious patient & both pulse & breathing present
Unconscious patient with pulse present & breathing absent
Unconscious patient with pulse absent & breathing present

Correct Answer - B
Explanation

ABCs

The initial step in the evaluation of an unconscious patient is to evaluate for the basic signs of life. The American
Heart Association recommends examining for a pulse, followed by assessing airway patency and breathing
pattern. If the patient does not have a pulse or does not have a regular breathing pattern, basic life
support/advanced cardiovascular life support is indicated.

91
Q. 120 - Identify the position?

A. Head tilt chin lift B. Jaw thrust


C. Manual inline stabilization D. none

Correct Answer - A
Explanation
Head tilt–chin lift

● Tilt the patient's head back by pushing down on the forehead.


● Place the tips of your index and middle fingers under the chin and pull up on the
mandible (not on the soft tissues). This lifts the tongue away from the posterior pharynx
and improves airway patency.

Q. 121 - Antibiotic of choice for patient posted for elective cardiac surgery?

A. Cefazolin B. Penicillin G
C. Azithromycin D. Erythromycin

Correct Answer - A
Explanation
Cefazolin is a first-generation cephalosporin with a broad spectrum of activity against many common pathogens
encountered in surgical site infections. It is typically administered intravenously within one hour before the
surgical incision.

Cefazolin (1–2 g bolus at induction possibly repeated after cardiopulmonary bypass) remains the standard for
antibiotic prophylaxis in cardiac surgery.

92
Internal Medicine

Q. 122 - The patient presents with BP of 160/110 mm Hg & on CT head SAH is detected. What is the next step
in managing this patient

A. Nimodipine B. Conventional angiography for aneurysm


C. Urgent surgical intervention D. IV Mannitol

Correct Answer - B
Explanation
The management of a patient with subarachnoid hemorrhage (SAH) and hypertension involves controlling the
blood pressure to prevent further bleeding and cerebral damage. The goal of blood pressure management is to
achieve a systolic blood pressure (SBP) of less than 140 mmHg.

Conventional angiography is not the next step in managing every patient with SAH and hypertension. The initial
management typically involves the following steps:

Ensure adequate airway, breathing, and circulation (ABCs).


Control hypertension: The patient may need medications to lower blood pressure, such as intravenous (IV)
nicardipine or labetalol. If the blood pressure remains elevated, additional medications may be required.

Evaluate the need for neurosurgical intervention: Patients with suspected aneurysmal SAH require urgent
neuroimaging with CT or MRI. If a ruptured aneurysm is identified, neurosurgical intervention is required to
prevent rebleeding and to secure the aneurysm.

Admit the patient to the intensive care unit (ICU) for monitoring and management of potential complications.

93
Conventional angiography may be indicated in patients with suspected cerebral aneurysms to identify the
location and size of the aneurysm, and to plan for neurosurgical or endovascular intervention.

Q. 123 - Which Murmur is shown below

A. MR B. AS
C. MS D. AR

Correct Answer - A
Explanation
The mitral regurgitation murmur is a holosystolic (heard throughout systole) murmur that typically begins after
S1 (the first heart sound) and ends before S2 (the second heart sound). The intensity of the murmur may
increase as the left ventricular pressure rises during systole. The waveform of the murmur may be a plateau
shape, meaning it has a relatively constant intensity throughout systole, or it may have a
crescendo-decrescendo shape, meaning the intensity of the murmur increases and then decreases over the
course of systole. The murmur may also have a blowing quality and may be heard best at the apex of the heart,
with radiation to the axilla.

Q. 124 - A 10 year old boy is admitted with features of severe dehydration nausea & vomiting. Urine sugar is
3+ & RBS= 550 mg/dl. ABG shows pH of 7.1 & HCO3= 7 mmol/L. Line of management as per ISPAD guidelines
is?

A. 10 ml per kg of iv fluids & 0.1 unit/kg/hr bolus


B. 10 ml per kg of iv fluids & after 1 hour of fluid 0.1 unit/kg/hr of insulin
C. 20 ml per kg of iv fluids & 0.1 unit/kg/hr of insulin bolus
D. 20 ml per kg of iv fluids & after 1 hour of fluid 0.1 unit/kg/hr of insulin bolus

Correct Answer - B
Explanation
The patient's presentation of severe dehydration, nausea, and vomiting, along with high urine sugar and high
blood glucose levels, are indicative of diabetic ketoacidosis (DKA), which is a serious complication of
94
uncontrolled diabetes. The ABG results suggest the presence of metabolic acidosis, which is a common feature
of DKA.

The International Society for Pediatric and Adolescent Diabetes (ISPAD) guidelines recommend initial
management with intravenous fluids and insulin for patients with DKA. According to the guidelines, the initial
fluid bolus should be 10 ml/kg of isotonic saline over the first hour of treatment, followed by maintenance fluid
therapy with half-normal saline at a rate of 1.5-2 times the maintenance fluid requirement. The goal of fluid
therapy is to correct dehydration and restore intravascular volume, while avoiding fluid overload.

Once fluid resuscitation is underway, insulin therapy should be initiated at a dose of 0.1 unit/kg/hr as a
continuous infusion. The goal of insulin therapy is to lower blood glucose levels and promote the resolution of
metabolic acidosis. Blood glucose levels should be monitored closely, with a target range of 150-200 mg/dL for
children with DKA.

It may require electrolyte replacement to correct imbalances such as hypokalemia and hyponatremia, which can
occur as a result of osmotic diuresis and volume depletion.

Q. 125 - A 65-year-old patient who occasionally takes alcohol & is on aspirin therapy for 4 years is having
epigastric pain in the abdomen with hematemesis & melena. What is the likely diagnosis

A. PUD B. Mallory Weiss


C. Boerhaave syndrome D. Variceal bleed

Correct Answer - A
Explanation
The patient's symptoms of epigastric pain in the abdomen, hematemesis (vomiting of blood), and melena (dark,
tarry stools) are suggestive of a bleeding peptic ulcer, which can be caused or exacerbated by factors such as
alcohol consumption and long-term aspirin therapy. Therefore, PUD (peptic ulcer disease) is a likely diagnosis in
this case.

Q. 126 - A patient is surgically treated for a pituitary tumor. He is now having polyuria with high serum
osmolarity. Which of the following is correct about the treatment of this patient?

A. Desmopressin for life long basis B. Desmopressin for 2 weeks


C. Vaptans for life long basis D. Thiazides for 2 weeks
95
Correct Answer - B
Explanation
It is possible that the patient's polyuria and high serum osmolarity are due to the surgical removal of a pituitary
tumor, which can result in decreased production of antidiuretic hormone (ADH) and subsequent diabetes
insipidus. In this case, treatment with desmopressin may be appropriate.

Desmopressin is a synthetic form of ADH that can be used to replace the missing hormone in patients with
central diabetes insipidus. It works by increasing water reabsorption in the kidneys, which can help to reduce
urine output and increase urine concentration. Desmopressin can be administered orally, intranasally, or
intravenously.

Q. 127 -Lady develops severe anemia, fatigue & gum hyperplasia. P. smear is shown below. Which of the
following chromosomal translocation would be responsible

A. AML with t(15:17) B. AML with inv 16


C. AML with t(8:21) D. AML with mutated NPM1

Correct Answer - A
Explanation
The symptoms of severe anemia, fatigue, and gum hyperplasia are consistent with acute myeloid leukemia

(AML) and can be specifically indicative of the subtype of AML with t(15:17) chromosomal translocation.

The t(15:17) chromosomal translocation is a specific genetic abnormality that is found in a subtype of AML

called acute promyelocytic leukemia (APL). APL is characterized by the abnormal proliferation of immature

myeloid cells, which can lead to a deficiency of normal blood cells, including red blood cells, leading to severe

anemia and fatigue. Gum hyperplasia, or excessive growth of gum tissue, is a common feature of APL, and may

be due to the infiltration of leukemic cells into the gums.

96
Q. 128 - HIV positive patient with a CD4 count of 100 presents with the following lesions. Diagnosis

A. HIV with disseminated histoplasmosis B. HIV with disseminated cryptococcosis


C. HIV with Molluscum contagiosum D. HIV with P.jiroveci

Correct Answer - B
Explanation
The diagnosis of HIV with disseminated cryptococcosis is a possibility in this patient based on the clinical
presentation and CD4 count.

Cryptococcosis is a fungal infection caused by the organism Cryptococcus neoformans, which is found in soil and
bird droppings. In HIV-positive patients with low CD4 counts, such as this patient with a CD4 count of 100,
cryptococcosis can become a disseminated infection, affecting multiple organs including the skin, lungs, and
central nervous system.

Skin lesions are a common manifestation of disseminated cryptococcosis, and can present as reddish-brown
papules or nodules on the face, chest, and other areas of the body. These lesions may be accompanied by fever,
fatigue, weight loss, and other systemic symptoms.

Q. 129 - An alcoholic patient presents with painful & swollen 1st MTP. Aspiration reveals increased leukocytes
>2000 cells/ cumm. There is no history of travel dysuria, dysentery, sexual contact in patients. BUN & serum
uric acid is normal. Clinical diagnosis is

A. Acute gouty arthritis B. Septic arthritis


C. Reactive arthritis D. Pseudogout

Correct Answer - A
Explanation
97
Based on the clinical presentation and laboratory findings, the patient is likely experiencing an acute gouty
arthritis flare.

Gout is a type of arthritis that is caused by the buildup of uric acid crystals in the joints, which can lead to
inflammation and severe pain. It is more common in patients who have a history of alcohol abuse, as alcohol
can increase uric acid production and decrease its excretion from the body.

The first metatarsophalangeal joint (1st MTP) is a common site of gouty arthritis flare-ups, and the presence of
pain, swelling, and increased leukocyte count in joint fluid are consistent with this diagnosis.

The normal BUN and serum uric acid levels do not rule out gout, as uric acid levels can be normal during an
acute gouty arthritis attack.

Q. 130 - The following congenital heart disease is seen due to a defect in which aortic arch

A. 2 B. 3
C. 4 D. 6

Correct Answer - D
Explanation
The 6th aortic arch gives rise to the ductus arteriosus and the pulmonary arteries, which will eventually
anastomose with arteries developing in the lungs.

Patent ductus arteriosus (PDA) is a persistent opening between the two major blood vessels leading from the
heart. The heart problem is present from birth. That means it is a congenital heart defect. An opening called the
ductus arteriosus is part of a baby's blood flow system in the womb.

Q. 131 - A 50-year-old laborer becomes unconscious & is brought to ER with a temp of 105 deg F with
decreased skin turgor. Which of the following will not be seen.

98
A. Sweating B. Hypotension
C. Tachypnea D. Red & hot skin

Correct Answer - A
Explanation
The given condition indicate heat stroke

Symptoms of heat stroke include:

● Confusion, altered mental status, slurred speech.


● Loss of consciousness (coma)
● Hot, dry skin or profuse sweating.
● Seizures.
● Very high body temperature.
● Fatal if treatment is delayed.

Q. 132 - A 75-year-old lady does not talk to anyone, sits silently and looks at the sky. On autopsy
hippocampus was examined and microscopic examination showed Lewy bodies. Diagnosis

A. PD B. AD
C. Huntington chorea D. Prion

Correct Answer - A
Explanation
Lewy body dementia is characterized by the abnormal buildup of proteins into masses known as Lewy bodies.
This protein is also associated with Parkinson's disease. People who have Lewy bodies in their brains also have
the plaques and tangles associated with Alzheimer's disease.

Risk factors:- A few factors seem to increase the risk of developing Lewy body dementia, including:
● Age. People older than 60 are at greater risk.
● Sex. Lewy body dementia affects more men than women.
● Family history. Those who have a family member with Lewy body dementia or Parkinson's disease are at
greater risk.

99
Q. 133 - Wife complains that her husband is not taking interest in daily activities. Tremors & rigidity is noticed
on examination. Lesion is located at

A. Prefrontal cortex B. Basal ganglia


C. Amygdala D. Insular cortex

Correct Answer - B
Explanation
Symptoms for basal ganglia lesion

● Movement changes, such as involuntary or slowed movements.


● Increased muscle tone.
● Muscle spasms and muscle rigidity.
● Problems finding words.
● Tremor.
● Uncontrollable, repeated movements, speech, or cries (tics)
● Walking difficulty.

Q. 134 - Alcoholic patient with loose motions a few days ago now presents with pain at the right
hypochondrium & USG Shows an abscess with 75 cc volume. Next best step?

A. PAIR B. Medical management


C. Surgical management D. Percutaneous drainage

Correct Answer - B
Explanation
Given case points towards the diagnosis of an amoebic liver abscess. Treatment would be medical management.
Antibiotics such as metronidazole (Flagyl) or tinidazole (Tindamax) are the usual treatment for liver abscesses.

A drug such as paromomycin or diloxanide must also be taken to get rid of all the ameba in the intestine and to
prevent the disease from coming back.

Q. 135 - AIDS positive with CD4 count of 150 cells/cumm. Chest infiltration is seen on CXR with mediastinal
adenopathy & per abdomen examination shows hepatosplenomegaly. BMA examination shows

A. Visceral leishmania B. Toxoplasmosis

100
C. Histoplasmosis D. P. Jiroveci

Correct Answer - C
Explanation
Mediastinal and hilar lymphadenopathy is a common condition in patients infected with HIV. It has been
previously reported to occur in up to 35–40% of patients infected with HIV. The broad range of aetiologies and
the non-specific clinical presentation represents a diagnostic challenge

The presence of mediastinal widening in association with subacute or chronic mediastinal lymphadenopathy
strongly suggests histoplasmosis. Cryptococcus can cause mediastinal lymphadenitis or mediastinitis, or both, in
immunocompromised hosts, mimicking lymphoma,101 and in HIV patients as part of acquired
immunodeficiency syndrome (AIDS), particularly in the immune reconstitution syndrome.

Q. 136 - Clonorchis sinensis causes

A. Bladder urinary B. Gallbladder


C. Biliary tract D. Stomach cancer

Correct Answer - C
Explanation
Clonorchis sinensis, often called the Chinese or liver fluke, can produce clonorchiasis in humans characterized
by inflammation and intermittent obstruction of the biliary ducts. In the acute phase, abdominal pain, nausea,
diarrhea, and eosinophilia can occur.

Q. 137 - HRP-2 Dipstick used for diagnosis of

A. Vivax B. Ovale
C. Falciparum D. Malariae

Correct Answer - C
Explanation

101
Diagnostic tools based on the dipstick principle for the detection of plasmodial histidine-rich protein 2 (HRP-2)
(ICT Malaria P.f. (R)) and parasite-specific lactate-dehydrogenase (pLDH) (OptiMal(R)), respectively, have
become available for the qualitative detection of falciparum malaria.

Q. 138 - Patient from Delhi with high-grade fever with petechiae & platelet count of 20k while TLC is 3K.
Diagnosis is

A. Malaria B. Dengue
C. Scrub typhus D. Typhoid

Correct Answer - B
Explanation
A typical person has a platelet count of between 150,000 and 250,000 per microlitre of blood. About 80 to 90
percent of patients with dengue will have levels below 100,000, while 10 to 20 percent of patients will see
critically low levels of 20,000 or less. Up to half of patients with dengue fever develop a characteristic rash. The
rash is variable and may be maculopapular or macular. Petechiae and purpura may develop as hemorrhagic
manifestations. Hemorrhagic manifestations most commonly include petechiae and bleeding at venipuncture
sites.

In the early febrile period of dengue fever, the majority have normal white blood cell count. Any change in the
values of total leucocyte count points toward the progression of the disease toward severity. There is
neutropenia in most of the patients with dengue fever.

Q. 139 - Hypertensive patient is taking multiple medications. His ECG is given. Which drugs are responsible
for this?

A. Spironolactone B. Thiazides
C. Prazosin D. Metoprolol

102
Correct Answer - A
Explanation
Spironolactone is used in combination with other medicines to treat high blood pressure (hypertension) and
heart failure. Lowering blood pressure can reduce the risk of strokes and heart attacks. Spironolactone is also
used to lessen the need for hospitalization for heart failure.

Q. 140 - Patient presents to the ER with the consumption of 100 tablets of aspirin. Management

A. Glucagon B. NAC
C. Urinary alkalinization D. HCN

Correct Answer - C
Explanation
Given scenario points towards the diagnosis of metabolic acidosis
Urine alkalinization is a treatment regimen that increases poison elimination by the administration of
intravenous sodium bicarbonate to produce urine with a pH ≥7.5.

The treatment of aspirin overdose, also known as salicylate toxicity, typically involves a combination of
supportive care, medications to reduce salicylate levels in the body, and monitoring for complications.

The specific treatments for aspirin overdose may vary depending on the severity of the overdose and the
patient's individual health status, but some common treatments include:

Activated charcoal: This may be given to absorb any remaining aspirin in the stomach and prevent further
absorption into the bloodstream.

Alkalinization of urine: This involves administering intravenous sodium bicarbonate to alkalinize the urine, which
helps to enhance the elimination of salicylates from the body.

Intravenous fluids: These are often given to maintain hydration and electrolyte balance.
Hemodialysis: In severe cases of aspirin overdose, hemodialysis may be required to remove salicylates from the
blood.

103
Monitoring and supportive care: This includes monitoring vital signs, oxygen saturation, and urine output, as
well as treating any complications that may arise, such as respiratory depression or metabolic acidosis.

Q. 141 - Patient with very high urea levels underwent hemodialysis. Post dialysis he develops seizures &
altered mentation. Which of the following will be used in management.

A. Mannitol B. Ethacrynic acid


C. Furosemide D. Bumetanide

Correct Answer - A
Explanation
Treatment of the dialysis disequilibrium syndrome
The treatment of disequilibrium after it has developed is aimed at reducing the intracranial pressure of the
patient. Standard maneuvers are to give mannitol or hypertonic saline to raise the blood osmolality and to
hyperventilate the patient.

Q. 142 - A 25-year-old lady with neck swelling presents with tremors & palpitations. She is a known case of
asthma & BP = 180/110 mmHg. ECG shows atrial fibrillation. Management

A. PTU B. Esmolol
C. Propranolol D. Diltiazem

Correct Answer - D
Explanation
In this case we ll first treat atrial fibrillation followed by other things.
Diltiazem is effective in treating AFIB-RVR by slowing conduction through the atrioventricular node (AV) and by
prolonging AV node refractoriness. When compared head-to-head with metoprolol or digoxin, diltiazem bolus
was superior in achieving ventricular rate control at 30 minutes post-administration

Q. 143 - Patient presents with fatigue & mid diastolic murmur with prominent A wave on JVP seen in

A. TS B. MS
104
C. TR D. AR

Correct Answer - A
Explanation
A Wave in JVP is an expression of normal atrial contraction.
Symptoms of tricuspid stenosis include a fluttering discomfort in the neck, fatigue, cold skin, and right upper
quadrant abdominal discomfort. Jugular pulsations are prominent, and a presystolic murmur is often heard at
the left sternal edge in the 4th intercostal space and is increased during inspiration. Diagnosis is by
echocardiography.

Q. 144 - Test for diagnosis of HSV Encephalitis

A. CSF PCR Viral load B. CSF Chick embryonal culture


C. CSF Tzanck smear D. CSF Culture

Correct Answer - A
Explanation
Herpes simplex encephalitis is the most common identified cause of sporadic viral encephalitis in the United
States. Early diagnosis is critical because treatment with the antiviral drug acyclovir dramatically decreases
morbidity and mortality. The use of polymerase chain reaction (PCR) techniques to amplify the genome of
herpes simplex virus (HSV) from cerebrospinal fluid (CSF) has become the diagnostic procedure of choice.
False-positive CSF HSV PCR results are rare when testing is performed in experienced laboratories. Negative CSF
HSV PCR results should always be interpreted in the context of the timing of specimen collection and the
likelihood of disease.

Negative CSF HSV PCR tests can occur within the first 72 hours of illness, with subsequent tests becoming
positive. Patients with HSV encephalitis will typically have a negative CSF HSV PCR after 14 days of acyclovir
treatment, and a persisting positive PCR should prompt consideration of additional or revised antiviral therapy.
Quantitative PCR testing provides information about HSV viral load in CSF, but the potential correlation of viral
load with prognosis or other clinical features of disease remains uncertain.

Q. 145 - Cement factory worker using heavy machinery & paint job complain of developing white fingers.
Possible etiology is
105
A. Asbestosis B. Silicosis
C. Vibration D. Paint & chemicals

Correct Answer - C
Explanation
Any vibrating tool that causes you a feel of tingling or numbness in your fingers after 5 minutes of continuous
use, could lead to Raynaud's phenomenon. Raynaud's phenomenon is also seen in typists and professional
pianists from repeated finger stress.

Raynaud's disease causes smaller arteries that supply blood flow to the skin to narrow in response to cold or
stress. The affected body parts, usually fingers and toes, might turn white then blue. Depending on the skin
color, these color changes may be harder or easier to see.

Q. 146 - In which of the following bilateral hemorrhage in the temporal lobe is seen

A. HSV Encephalitis B. Amoebic meningoencephalitis


C. Enterovirus meningitis D. None of the above

Correct Answer - A
Explanation
Herpes simplex virus type 1 causes an encephalitis in humans that is primarily restricted to the temporal lobe
and limbic system. The distribution of lesions suggests that the virus enters the brain from a single site and then
spreads transneuronal to infect connected structures.

Q. 147 - 30 year old with chronic Hep B & HBeAg positive with viral load 105 copies/ ml & doubling of SGPT
shall be treated with

A. Tenofovir >48 weeks


B. Lamivudine + Tenofovir >48 weeks
C. Pegylated interferon with Tenofovir for 6 months
D. Pegylated interferon with Lamivudine for 48 weeks

106
Correct Answer - A
Explanation
Treatment for chronic hepatitis B with Tenofovir for 48 weeks is a common approach to manage the infection.
In this case, the patient is HBeAg positive with a viral load of 105 copies/ml and a doubling of SGPT. These
factors indicate active viral replication and liver inflammation, which may increase the risk of liver damage over
time if left untreated.

Tenofovir is a highly effective antiviral medication that can suppress viral replication, reduce liver inflammation,
and lower the risk of liver complications. However, treatment may need to be continued beyond 48 weeks,
depending on the patient's response to therapy and the duration of treatment required to achieve viral
suppression.

Surgery

Q. 148 - Q SOFA score includes

A. PR/BP/Temp B. Temp/BP/GCS
C. RR/BP/GCS D. HR/BP/Temp

Correct Answer - C
Explanation
The qSOFA score is a simple score consisting of three items:
1. respiratory rate (RR) ≥ 22 breaths per minute,
2. altered mentation (Glasgow Coma Scale [GCS] < 15),
3. systolic blood pressure (SBP) < 100 mmHg.

Q. 149 - 30 year old patient with a 4 week H/O midline neck swelling which moves up on deglutition and
protrusion of tongue as image given below. Diagnosis will be?

107
A. Thyroglossal cyst B. Plunging Ranula
C. Branchial cyst D. Dermoid cyst

Correct Answer - A
Explanation
Thyroglossal duct cysts are the most frequently occurring congenital cervical anomalies
They often present as midline neck cysts closely associated with the hyoid bone.
Thyroglossal duct cysts typically present as mobile midline neck masses near the hyoid bone.

Q. 150 - A 23 year old medical student presented with exophthalmos, palpitation and heat intolerance as
image given below. Which antibody is expected to be high in this patient?

A. Against thyroid receptor B. Anti TPO


C. Antithyroglobulin D. Serum thyrotropin

Correct Answer - A
Explanation
Case of thyrotoxicosis
Thyrotoxicosis is a clinical state of inappropriately high levels of circulating thyroid hormones (T3 and/or T4) in
the body from any cause

Thyrotoxicosis results from thyroid hormone excess either from endogenous over-secretion of T3 and T4 or
from exogenous ingestion of synthetic thyroid hormone.
108
Q. 150 - A 45 year old patient operated laparoscopic inguinal hernia. Post operatively for 3 years he is having
pain & loss of sensation in lower leg. Most common nerve stuck in the fixator is?

A. IIioinguinal nerve B. ILiohypogastric nerve


C. Femoral nerve D. Lateral cutaneous nerve of thigh

Correct Answer - D
Explanation
During open hernia repair, the ilioinguinal, iliohypogastric, and the genitofemoral nerves are most commonly
injured, while the lateral femoral cutaneous nerve is more commonly injured during laparoscopic
herniorrhaphy

Q. 151 - Deep ring is a defect in which structure?

A. External oblique B. Internal oblique


C. Transversus abdominis D. Fascia transversalis

Correct Answer - D
Explanation
The deep inguinal ring, through which the round ligament or spermatic cord enters the inguinal canal, is a
defect in the transversalis fascia.
109
Q. 152 - A child's urine turns dark while standing. The probable cause is?

A. Type I tyrosinemia B. Congenital erythropoietic porphyria


C. Type II tyrosinemia D. Alkaptonuria

Correct Answer - D
Explanation
The three major features of alkaptonuria are dark urine or urine that turns dark on standing, ochronosis
(bluish-black pigmentation in connective tissue), and arthritis of the spine and larger joints.

Q. 153 - A patient presented with RTA. O/E he opens his eyes on painful stimuli, speaks inappropriate words
& shows withdrawals of limb on stimulation. What is his GCS score?

A. E2, V3, M4 B. E2, V4, M4


C. E3, V4, M3 D. E2, V4, M4

Correct Answer - A
Explanation

110
Q. 154 - Clonorchis sinensis is associated with which cancer?

A. HCC B. Cholangiocarcinoma
C. Gastric cancer D. Urinary bladder cancer

Correct Answer - B
Explanation
In humans, these flukes dwell in the bile ducts and can cause bile duct cancer. The parasites closely related to
risk of developing bile duct cancer are C. sinensis and O. viverrini.

Q. 155 - Good prognostic factor for BIRADS V breast cancer is

A. P53 +ve B. BRCA 1


C. Ki67 D. ER +v

Correct Answer - D
Explanation

Q. 156 - Direct inguinal hernia which part is weak

A. Conjoined tendon B. Pectinate ligament


C. Lacunar ligament D. Poupart ligament

111
Correct Answer - A
Explanation
The conjoint tendon makes up the main part of the medial portion of the posterior wall of the inguinal canal.
The conjoint tendon has an essential role in protecting a weak area in the abdominal wall in which a weakening
of the conjoint tendon may lead to a direct inguinal hernia

Q. 157 - A 56 year old chronic smoker presented with weight loss and cough for more than 1 year. He is
having hypercalcemia. On biopsy there are atypical cells with hyperchromatic nuclei. What is the diagnosis?

A. SCC B. Adenocarcinoma
C. Small cell cancer D. Atypical large cell

Correct Answer - A
Explanation
Squamous cell carcinoma, the second most common form of skin cancer, is caused by the cumulative exposure
of skin to UV light.

This condition has precursor lesions called actinic keratosis, exhibits tumor progression and has the potential to
metastasize in the body.

Squamous carcinomas are the most common malignancies that cause humoral-mediated hypercalcemia of
malignancy.

Q. 158 - A 45 year old alcoholic patient with 1week H/O diarrhea & right upper abdominal pain with 75 cc
liver abscess. What is the first line of treatment?

A. Percutaneous drainage B. Medical treatment


C. Surgical drainage D. PAIR

Correct Answer - C
Explanation
First line of treatment will be surgical drainage

Q. 159 - What is the CEAP classification for varicose veins with eczema?

112
A. C2 B. C3
C. C4 A D. C4B

Correct Answer - C
Explanation

Q. 160 - A 4 month old child is presented with jaundice and pale stool. On investigations ductal dilatation and
biliary plug was found. ALP is high while AST & ALT are normal. Biopsy showed ductal proliferation and
fibrosis of ducts. Most probable diagnosis is?

A. Criger Najjar syndrome B. Dubin Johnson syndrome


C. Biliary atresia D. HCC

Correct Answer - C
Explanation
113
Biliary atresia is an obstructive cholangiopathy of unknown etiology involving both the intrahepatic and
extrahepatic bile ducts.

It presents in the neonatal period with persistent jaundice, clay-colored stools, and hepatomegaly
The histological examination of biliary atresia specimens shows variable liver fibrosis, bile duct proliferation, bile
duct plugging, cholestasis, inflammatory cell infiltration.

Among all the features, bile duct proliferation is a highly sensitive and specific marker for biliary atresia.
As alkaline phosphatase levels are elevated in children due to bone remodeling

Q. 161 - A chronic alcoholic had high NADH: NAD ratio, with normal urea. Which of the following is expected
in this individual?

A. Low lactate B. High glucose


C. High urate D. Low urate

Correct Answer - C
Explanation
Effects of Chronic Alcohol Use
● Epigenetic Modifications
● Alters DNA Methylation and Histone Acetylation Patterns
● Alters the ratio of NADH:NAD+
● Alters carbohydrate metabolism
● Promotes the formation of Reactive Oxygen Species
● Lipid and protein oxidation
● DNA Damage – Double strand breaks
● Contributes to Cellular Apoptosis
Increase in NADH/NAD+ Ratio-
● In the Liver, the increase in cytosolic NADH favors the formation of lactate resulting in lactic acidosis this
can cause Alcoholic hypoglycemia

Q. 162 - True statement for the following condition shown in image

114
A. Caused by mixed bacterial infection B. Bladder diversion required
C. B/L orchidectomy needed D. Anti gas gangrene serum should be given

Correct Answer - A
Explanation
Fournier's gangrene
Fournier's gangrene (FG) is a fulminant form of infective necrotising fasciitis of the perineal, genital, or perianal
regions, which commonly affects men. Cultures from the wounds commonly show poly microbial infections by
aerobes and anaerobes, which include coliforms, klebsiella, streptococci, staphylococci, clostridia, bacteroides,
and corynebacteria

Q. 163 - A 30 year old patient presented to hospital with abdominal pain, weight loss and sterile pyuria as
given in image. Diagnosis will be?

A. Putty kidney B. Pyelonephritis


C. Nephrocalcinosis D. Staghorn calculus

Correct Answer - A
Explanation
115
A putty kidney refers to a pattern of renal calcification associated with renal tuberculosis conventionally
described on plain radiography. The calcification can be large, round or oval, dense or very homogeneous and
ground glass-like, representing calcified caseous tissue within dilated calyces

Q. 164 - A patient of chronic pancreatitis presented to casualty with upper abdominal pain. Pancreatic duct is
10 mm in size with calculi inside the duct. What is the best treatment plan for this patient?

A. Gastrojejunostomy B. Distal pancreatectomy


C. Longitudinal pancreaticojejunostomy D. Total pancreatectomy

Correct Answer - C
Explanation
In chronic pancreatitis, longitudinal pancreaticojejunostomy (LPJ) is the treatment of choice for patients with a
dilated main pancreatic duct (MPD) without an inflammatory mass in the pancreatic head. This procedure is
associated with low morbidity and mortality rates.

Q. 165 - A patient with a history of trauma 5 years back has following feature as given below in image. What
is it?

A. Keloid B. Thrombophlebitis
C. Hemangioma D. Neurofibroma

Correct Answer - A
Explanation
Keloids result from abnormal wound healing in response to skin trauma or inflammation.

116
Predisposed individuals may develop a keloid following any level of skin trauma including surgery, piercings,
acne, tattooing, insect bites, burns, lacerations, abrasions, vaccinations, and any other process resulting in
cutaneous inflammation. Increased tension in a wound also may contribute to keloid formation.

Q. 166 - A 55 year old alcoholic patient presented with weight loss. Investigations showed increased AFP and
normal AST and ALT. Conjugated bilirubin is also high. Most probable diagnosis is?

A. Cholangiocarcinoma B. HCC
C. Adenoma D. Alcoholic hepatitis

Correct Answer - B
Explanation
Hepatocellular carcinoma (HCC) is a primary tumor of the liver.
Liver function tests including bilirubin, alanine aminotransferase (ALT), aspartate aminotransferase (AST),
alkaline phosphatase (ALP), and albumin may be elevated on the initial evaluation.

Q. 167 - Patient c/o loss of sensation or callosity with the image given below. What is the Diagnosis?

A. Arterial ulcer B. venous ulcer


C. Trophic ulcer D. Malignant ulcer

Correct Answer - C
Explanation
Pressure ulcer (Trophic ulcer) caused by external trauma to a part of the body that is in poor condition because
of disease, vascular insufficiency or loss of afferent nerve fibers

117
Q. 168 - A 5 year old child presented with bleeding P/R and a rectal polyp. Biopsy showed dilated glands and
presence of mucin with inflammatory cells. Most probable diagnosis?

A. Hamartomatous polyp B. Adenomatous polyp


C. Choriostoma D. Adenocarcinoma

Correct Answer - A
Explanation
The hamartomatous polyposis syndromes are a heterogeneous group of disorders that share an
autosomal-dominant pattern of inheritance and are characterized by hamartomatous polyps of the
gastrointestinal tract.

These syndromes include juvenile polyposis syndrome, Peutz–Jeghers syndrome and the PTEN hamartoma
tumor syndrome.

Q. 169 - Best treatment for a case of NaOH ingestion with complete dysphagia is?

A. Feeding jejunostomy B. Esophagojejunostomy


C. Gastrojejunostomy D. Stenting

Correct Answer - A
Explanation
Feeding jejunostomy refers to a surgically inserted tube, preferably in the proximal jejunum, to provide enteral
nutrition or administer medications.

118
Q. 170 - The patient shown in image is having dyspnoea. Which of the following is false statement about this
condition?

A. Hyperbaric O2 has no role


B. Debridement may be required daily
C. This is known as Meleney gangrene
D. Streptococci is the main organism which becomes polymicrobial later on

Correct Answer - A
Explanation
Meleney’s gangrene is a rare, rapidly spreading destructive subcutaneous tissue infection which most
commonly occurs at post-surgical sites and has a mortality rate of as high as 30-40% (up to 90% in diabetics).

Meleney's gangrene mostly affects the skin and subcutaneous tissue


Meleney's synergistic gangrene is caused by S aureus and streptococcus organisms
The ulcers that form at the center of the lesion are usually covered by a black eschar and encircled by a
gangrenous margin It requires Debridement may be daily

Q. 171 - A patient came in with fever. Mass at glans penis and has verrucous growth as shown in image
below. What is the treatment of choice?

119
A. Partial penectomy B. Podophyllin local application
C. CO2 laser D. 5 Fu cream application

Correct Answer - A
Explanation
Verrucous cancer
Verrucous carcinoma is a relatively uncommon, locally aggressive, clinically exophytic, low-grade, slow-growing,
well-differentiated squamous cell carcinoma with minimal metastatic potential.

Surgical excision and Mohs micrographic (MMS) surgery represent the treatments of choice for cutaneous
verrucous carcinomas.

Verrucous carcinoma can often be treated with laser therapy, Mohs surgery, wide excision, or cryotherapy. A
partial penectomy can also be done.

Q. 172 - A patient with cirrhosis presents with bleeding. Which anastomosis is prone to bleeding in this case?

A. Left gastric and esophagus vein B. Superior rectal with sigmoid vessel
C. LGV with phrenic veins D. Mid Colic and Superior colic

Correct Answer - A
Explanation
120
The left gastric vein (LGV) anastomoses with the esophageal veins, which in turn drain into the azygos vein (AV)
The direction of collateral flow is always to bypass the occluded portion of the vessel. When vascular
obstruction is intrahepatic, collateral vessels drain away from the liver (hepatofugal collateral circulation). When
the obstruction is extrahepatic, the collateral circulation usually develops toward the portal vein beyond the site
of obstruction and thus drains toward the liver (hepatopetal collateral circulation).

Q. 173 - A patient is present with nodular swelling on cheek with mixed consistency as shown below. What is
the diagnosis?

A. Parotid tumor B. Dermoid tumor


C. Sebaceous cyst D. Tumor of jaw

Correct Answer - A
Explanation
A tumor of the parotid salivary gland arises from an overgrowth of cells. “There are different types of tumors,
but the most common type is pleomorphic adenoma, a slow-growing tumor
Patients present with a fullness or swelling of the cheek or an asymmetry in the mouth.

Q. 174 - Most common complication after the first major vessel ligation during APR (abdominal perineal
resection) is?

A. Sympathetic- bladder dysfunction and impotency


B. Parasympathetic- bladder dysfunction and erectile dysfunction
C. Sympathetic- loss of perineal sensation and impotency
D. Sympathetic- bladder dysfunction and retrograde ejaculation

Correct Answer - A
Explanation
121
Perineal wound problems after abdominoperineal resection (APR) in the context of anal and rectal cancers are
frequent.

Bladder dysfunction following colorectal surgery may be related to extirpative procedures in the region of the
pelvic autonomic plexus.
Sexual dysfunction following surgery for rectal cancer is also common.

Q. 175 - Pain in parotid tumor due to involvement of which structure

A. Auriculotemporal B. Facial
C. Investing layer of Deep cervical fascia D. Hypoglossal nerve

Correct Answer - A
Explanation
The parotid gland receives sensory and autonomic innervation. The autonomic innervation controls
the rate of saliva production.

Sensory innervation is supplied by the auriculotemporal nerve (gland) and the great auricular nerve
(fascia).

122
The parasympathetic innervation to the parotid gland has a complex path. It begins with the
glossopharyngeal nerve (cranial nerve IX). This nerve synapses with the otic ganglion (a collection
of neuronal cell bodies). The auriculotemporal nerve then carries parasympathetic fibres from the
otic ganglion to the parotid gland. Parasympathetic stimulation causes an increase in saliva
production.

Sympathetic innervation originates from the superior cervical ganglion, part of the paravertebral
chain. Fibres from this ganglion travel along the external carotid artery to reach the parotid gland.
Increased activity of the sympathetic nervous system inhibits saliva secretion, via vasoconstriction.

Pediatrics

Q. 176 - Black urine in infants caused by

A. Homocysteinemia B. Alkaptonuria
C. Phenylketonuria D. None

Correct Answer - B
Explanation
Alkaptonuria, or black urine disease, is a very rare inherited disorder that prevents the body fully breaking down
two protein building blocks (amino acids) called tyrosine and phenylalanine. It results in a build-up of a chemical
called homogentisic acid in the body.

Q. 177 - Images of BM aspiration (A and peripheral smear (B) from a child with pancytopenia shown below.
Which enzyme is deficient in this condition

123
A. Glucose 6 phosphatase B. Glucocerebrosidase
C. Hexosaminidase D. Alpha galactosidase

Correct Answer - B
Explanation
Gaucher disease is a rare, inherited metabolic disorder in which deficiency of the enzyme glucocerebrosidase
results in the accumulation of harmful quantities of certain fats (lipids), specifically the glycolipid
glucocerebroside, throughout the body especially within the bone marrow, spleen and liver.

Q. 178 - Best sign of adequate infant growth in an infant with 2.8 kg birth weight

A. Weight gain of 300 gm/ month till 1 year B. Increase in length of 25 cm in 1st year
C. Anterior fontanelle closure by 6 month D. Weight under 75% & length under 25%

Correct Answer - B
Explanation
At birth, an average Indian boy weighs 3 to 3.2kg at term and a girl weighs 2.8 to 3.2kg. In the first three
months, weight gain is faster with an average of 800-1000g per month. Over the next three months, it slows
down to 600-800g and between 6-12 months, it is not more than 500g usually.

Q. 179 -A 4 year old child was brought with episodes of icterus & pallor. What is the probable diagnosis &
investigation done?

A. Autoimmune hemolytic anemia, direct Coombs B. Autoimmune hemolytic anemia


C. Hereditary Spherocytosis osmotic fragility test D. None

Correct Answer - C
Explanation

124
The osmotic fragility test is useful for diagnosis of hereditary spherocytic hemolytic anemia. Spherocytes are
osmotically fragile cells that rupture more easily in a hypotonic solution than do normal RBCs.

Q. 180 - A 10 year old boy presented with C/o recurrent bleeding. There was also past history of joint pains.
No mucosal bleeding. Which coagulation factor deficiency can be responsible?

A. Factor 8 & 9 B. Factor 3 & 4


C. Vwf D. Lupus anticoagulant

Correct Answer - A
Explanation
The osmotic fragility test is useful for diagnosis of hereditary spherocytic hemolytic anemia

Q. 181 - Chloride level is tested in which disease?

A. Cystic fibrosis B. Gaucher's disease


C. Osteogenesis imperfecta D. None

Correct Answer - A
Explanation
A sweat test measures the amount of chloride in sweat. Chloride is part of the salt that's found in sweat. The
test can diagnose cystic fibrosis (CF) because people with CF have higher levels of chloride in their sweat. CF is
an inherited disease that can affect people of all ages.

Q. 182 - A 10 year old child presented with tachypnea and severe dehydration. He had a random blood
glucose of 550 mg/dl, urine glucose 3+. The blood gas showed pH 7.1, bicarbonate of 7 mmol/L. What is the
appropriate management?

A. Stabilize, NS 10 ml/kg followed by insulin 0.1 U/Kg/hr


B. Stabilize, NS 10 ml/Kg followed by insulin 0.2 U/Kg/hr
C. Stabilize, NS 20 ml/kg followed by insulin 0.1 U/Kg/hr
D. Stabilize, NS 20 ml/kg followed by insulin 0.2 U/Kg/hr
125
Correct Answer - A
Explanation
Need to stabilize the patient
Patient needs to given NS 10 ml/kg followed by insulin 0.1 U/Kg/hr

Q. 183 - Gum bleeding in scurvy is due to

A. Defect in collagen synthesis B. Increased keratinization of epithelium


C. Inhibition of clotting factors D. Low calcium

Correct Answer - A
Explanation
Vitamin C is functionally most relevant for the triple-helix formation of collagen; a vitamin C deficiency results in
impaired collagen synthesis.

Defective collagen synthesis leads to defective dentine formation, hemorrhaging into the gums, and loss of
teeth. Hemorrhaging is a hallmark feature of scurvy and can occur in any organ. Hair follicles are one of the
common sites of cutaneous bleeding.

Q. 184 - A child was brought with some bony deformities as shown below. It is caused by which deficiency

A. Vitamin A B. Vitamin D
C. Vitamin C D. Vitamin K

Correct Answer - B
Explanation

126
Vitamin D deficiency can lead to a loss of bone density, which can contribute to osteoporosis and fractures
(broken bones). Severe vitamin D deficiency can also lead to other diseases. In children, it can cause rickets.
Rickets is a rare disease that causes the bones to become soft and bend.

Obstetrics / Gynecology

Q. 185 - A 25-year-old pregnant female came for regular visit. She delivered twins 4 years ago. Calculate the
gravida & para.

A. G2P2 B. G2P1
C. G3P2 D. G3P1
Correct Answer - B
Explanation
Gravida and para are terms used to describe a woman's obstetric history:
G- Gravida refers to the total number of times a woman has been pregnant, including the current pregnancy.

127
P- Para refers to the number of pregnancies that have reached 20 weeks of gestation or more and resulted in
either a live birth or a stillbirth.

Based on the information provided, the woman has delivered twins 4 years ago, but it is not mentioned
whether she has been pregnant since then. Assuming that she has not been pregnant since then, the gravida
and para can be calculated as follows:

Gravida: 2 (including the current pregnancy)


Para: 1 (twin delivery 4 years ago)
Therefore, the woman's gravida is 2 and her para is 1.

Q. 186 - A primigravida female delivered a baby girl. She is a known case of bronchial asthma. There is
profuse bleeding after delivery. Which of the following drugs should not be used to control bleeding?

A. Carboprost B. Latanoprost
C. Misoprostol D. Oxytocin

Correct Answer - A
Explanation
Carboprost is a medication used to control postpartum hemorrhage (excessive bleeding after delivery) by
promoting uterine contractions. However, there are certain situations in which it should not be used, such as in

patients with a history of asthma or other respiratory conditions, as it can cause bronchoconstriction and
worsen respiratory symptoms.

In the case of the primigravida female with bronchial asthma who has delivered a baby girl and is experiencing
profuse bleeding after delivery, carboprost should not be used to control bleeding. Instead, alternative
medications and interventions should be considered, such as oxytocin, misoprostol, or uterine massage. The
patient's asthma should also be managed appropriately to avoid exacerbations during the postpartum period.

Q. 187 - In the cancer cervix, neoadjuvant therapy given along with

A. Along with surgery B. Along with radiotherapy


C. After radical surgery D. Before radical surgery
128
Correct Answer - D
Explanation
Neoadjuvant therapy is a type of cancer treatment that is given before the main treatment (usually surgery)
with the aim of shrinking the tumor and improving the effectiveness of the subsequent therapy. In the case of
cervical cancer, neoadjuvant therapy may be used before radical surgery (removal of the cervix and surrounding
tissues) sometimes.

Neoadjuvant therapy for cervical cancer typically involves chemotherapy, with or without radiation therapy. If a
large tumor, locally advanced, or has spread to nearby lymph nodes, and when there is a higher risk of
recurrence or metastasis after surgery alone. Neoadjuvant therapy can also be used to downstage the tumor,
making it easier to remove during surgery and potentially improving the chance of a cure.

Q. 188 - A 24-year-old female presents with 36 w + 6 days gestation. On examination the fetal was in a
transverse lie with adequate liquor, and the placenta was in a normal position. The weight of the baby was
2.3 kg. What is the next step in management?

A. Emergency cesarean B. External cephalic version


C. Vacuum delivery D. Expectant management

Correct Answer - B
Explanation
External cephalic version (ECV) is a procedure that is used to attempt to turn a baby from a breech or transverse
lie to a head-down position before labor starts. In this case, since the baby is in a transverse lie at 36 weeks and
6 days gestation, ECV may be considered as the next step in management.

ECV is typically performed by a healthcare provider who applies pressure to the mother's abdomen to manually
turn the baby. The procedure is usually done in a hospital setting, and the mother and baby are monitored
closely throughout the procedure to ensure that there are no complications.

Before ECV is attempted, the healthcare provider will usually perform an ultrasound to confirm the position of
the baby, assess the amount of amniotic fluid, and check the location of the placenta. In this case, since the
129
ultrasound shows that the placenta is in a normal position and there is adequate amniotic fluid, ECV may be
considered a viable option.

Q. 188 - A pregnant woman visits the OPD after 2 months of amenorrhea. She was found to be 7 weeks
pregnant & anemic. On examination Hb was 9gm%. The iron supplement has to be taken from which week

A. 8-10 weeks B. From 20 weeks


C. From 14 weeks D. 10-12 weeks

Correct Answer - C
Explanation
In this case, the woman is 7 weeks pregnant and anemic, with a hemoglobin level of 9 gm%.
Iron supplementation is given during pregnancy is typically recommended to prevent and treat anemia. The
American College of Obstetricians and Gynecologists (ACOG) recommends a daily dose of 30 mg to 60 mg of
elemental iron for pregnant women, starting at 12-16 weeks of gestation. However, in this case, since the
woman is already anemic, the healthcare provider may choose to start iron supplementation earlier, at 7 weeks
of gestation.

Q. 189 - A 30-year-old patient who is on infertility treatment with HMG. She developed abdominal pain, and
dyspnea. What is the condition called?
A. Ovarian hyperstimulation syndrome B. PCOD
C. Theca Lutein cyst D. Hypogonadotropic hypogonadism

Correct Answer - A
Explanation
Ovarian hyperstimulation syndrome (OHSS) is a medical condition that can occur in some women who undergo
fertility treatment with medications such as human menopausal gonadotropin (HMG). It is caused by an
excessive response to ovarian stimulation and is characterized by enlarged ovaries, fluid accumulation in the
abdominal cavity, and sometimes in the chest, and an increased risk of blood clots. Symptoms can include
abdominal pain, bloating, nausea, vomiting, and shortness of breath. OHSS can be mild, moderate or severe,
and severe cases require medical attention.

Q. 190 - Of the following surgeries posted in OT, all can be done by hysteroscope except:

A. Asherman syndrom B. Septum resection’


130
C. Subserous myoma D. Tubal ligation

Correct Answer - c
Explanation
The treatment of subserous myomas depends on their size, location, and symptoms.
Small subserous myomas that do not cause any symptoms may not require any treatment, but suggested
monitoring them with periodic ultrasounds to check for any changes.

If subserous myomas cause symptoms such as pain or pressure, or if they are causing infertility or other
problems, treatment options may include:
1. Hysteroscopic myomectomy: A hysteroscope is used to remove small subserous myomas that protrude into
the uterine cavity. This is a minimally invasive procedure that is done as an outpatient and has a short
recovery time.
2. Laparoscopic myomectomy: This is a minimally invasive surgery that uses small incisions to remove
subserous myomas that are larger or that extend deeper into the myometrium. This is done under general
anesthesia and may require a brief hospital stay.
3. Open surgery: This is a traditional surgical approach that may be necessary for very large subserous
myomas or those that are deeply embedded in the myometrium. This is done under general anesthesia
and requires a longer hospital stay and recovery time.
4. Observation: If subserous myomas are small and not causing any symptoms, your doctor may suggest
monitoring them with periodic ultrasounds.

Q. 191 - A 16-year-old girl with primary amenorrhea presents with cyclical abdominal pain with the
suprapubic bulge. On examination, there is swelling along the entire vagina. What is the most likely diagnosis

A. Complete vaginal atresia B. Imperforate hymen


C. Total Mullerian agenesis D. Transverse vaginal septum

Correct Answer - B
Explanation
Imperforate hymen is a possible diagnosis in this case. An imperforate hymen is a congenital disorder where the
hymen completely covers the vaginal opening, blocking menstrual blood from leaving the body. This can cause
cyclical abdominal pain due to the buildup of menstrual blood, as well as a visible swelling of the vagina.

131
Treatment typically involves a surgical incision to create an opening in the hymen to allow menstrual blood to
flow out.

Q. 192 - A 23-year-old female brought by her mother-in-law to the OPD with the complaint that she is not
able to conceive even after 6 months of Cohabitation. What is the next step of management?

A. Hysterosalpingography B. Revisit after 6 months


C. Husband semen analysis D. Hysterectomy

Correct Answer - B
Explanation
In general, current guidelines recommend that a couple should seek evaluation for infertility if they have been
trying to conceive for 12 months or more without success. However, in certain circumstances, such as advanced
maternal age or a known history of infertility, evaluation may be initiated after 6 months of trying. Therefore, if
there are no other concerning factors or red flags, it may be appropriate to advise the couple to continue trying
for another 6 months before pursuing further evaluation or intervention.

Q. 193 - A woman with cervical cancer was recently treated by radical surgery. What is the best advice given
to her 14-year-old daughter?

A. HPV Vaccine B. BRCA testing


C. PTEN Testing D. Cervical biopsy

Correct Answer - A
Explanation
Advising the HPV vaccine to the 14-year-old daughter of a woman with cervical cancer is appropriate. HPV
(Human papillomavirus) infection is the primary risk factor for cervical cancer, and the HPV vaccine can prevent
the infection. The vaccine is most effective when given before exposure to the virus, which is why it is typically
recommended for girls and boys aged 11-12 years, but it can be given up to age 45. The Centers for Disease
Control and Prevention (CDC) recommends that all girls and boys receive the HPV vaccine series at age 11-12
years.

132
Q. 193 - A 27-year-old female presents with 2 months of amenorrhea with the previous 3 regular cycles. On
USG, left adrenal mass of 3 x 2.5 months. Her HCG was 2800 miu/ml. There is no fetal pole in the uterus and
no cardiac activity. What is the next step of management?

A. Oral methotrexate B. Multiple doses of methotrexate with leucovorin


C. Single dose of methotrexate injection D. Two dose of methotrexate injection

Correct Answer - C
Explanation
The management of a patient with an adnexal mass and a positive pregnancy test depends on several factors,
including the size and appearance of the mass and the level of the beta-hCG. In this scenario, the beta-hCG level
is elevated and there is no evidence of an intrauterine pregnancy, indicating a possible ectopic pregnancy.
However, the presence of an adrenal mass is concerning for a non-gynecologic pathology, such as an adrenal
tumor, which may also secrete beta-hCG. Therefore, further workup is necessary to confirm the diagnosis.

The first step would be to repeat the beta-hCG level after 48 hours to determine if it is increasing or decreasing.
If it is increasing, this would support the diagnosis of an ectopic pregnancy and treatment with methotrexate or
surgical intervention would be required. However, if the beta-hCG level is decreasing, the diagnosis of an
ectopic pregnancy would be less likely and further evaluation of the adrenal mass would be necessary. This may
include imaging studies such as CT or MRI

Q. 194 - A woman with 32 weeks of gestation presents with the chief complaint of difficulty in breathing &
walking. Her fetal movements were normal. On examination the abdomen is tense but not tender, symphysis
fundal height is 41cm. Fetal parts are not easily felt. What is the likely diagnosis?
A. Polyhydramnios B. Oligohydramnios
C. Maternal-fetal ascites D. Stillbirth
Correct Answer - A
Explanation
Polyhydramnios is a condition characterized by excessive amniotic fluid accumulation, which can lead to
increased tension and pressure in the uterus, causing discomfort and difficulty in breathing and walking. The
symphysis fundal height measurement of 41 cm suggests a large-for-gestational-age fetus, which is common in
cases of polyhydramnios. The absence of fetal parts easily felt on examination may indicate that the fetus is
floating in the excess amniotic fluid.

133
Further evaluation, including ultrasound, is required to confirm the diagnosis and determine the underlying
cause of polyhydramnios. Treatment options depend on the severity and cause of polyhydramnios and may
include close monitoring, amnioreduction, and delivery if necessary.

Q. 195 - A 17-year-old girl with primary amenorrhea has breast development but the absence of pubic &
axillary hair. There are bilateral inguinal masses felt. On USG examination there is the absence of the uterus,
fallopian tube & ovaries. Which of the following best describes the scenario?

A. PCOS B. Turner Syndrome


C. Complete AIS D. Hypergonadotropic hypogonadism

Correct Answer - C
Explanation
The scenario described is most consistent with complete Androgen Insensitivity Syndrome (AIS). In this
condition, the person is genetically male (XY) but is insensitive to androgens (male hormones) and has external
female genitalia. They have breast development but no pubic or axillary hair. The absence of the uterus,
fallopian tube, and ovaries is due to the failure of Müllerian ducts to develop, which occurs in individuals with
XY chromosomes. The presence of bilateral inguinal masses is likely to be testes that failed to descend during
embryonic development.

Q. 196 - A 40 year old woman was scheduled for a total laparoscopic hysterectomy. She was concerned about
the procedure & enquired the gynecologist about laparoscopic hysterectomy. What is the disadvantage of
total laparoscopic hysterectomy?

A. Increase blood loss B. Long hospital stay


C. Increase postoperative pain D. Increased chances of Bladder injury

Correct Answer - D
Explanation
Benefits of laparoscopic hysterectomy:

1. Smaller Incisions hence post operative pain is lesser.


2. The hospital stay after laparoscopic surgery may be shorter
3. The patient may be able to return to normal activities sooner.
134
4. The risk of infection is lower.

Disadvantages of laparoscopic hysterectomy:

1. It often takes longer to perform laparoscopic surgery compared with abdominal or vaginal surgery. The
longer you are under general anesthesia, the greater the risks for certain complications.
2. Also, there is an increased risk of bladder injury in this type of surgery.

Q. 197 - A 53-year-old patient was diagnosed with cervical cancer. On examination, a necrotizing lesion on
the cervix extended to the parametrium. What is the management of this stage?

A. Surgery B. Chemoradiation
C. Biopsy D. Surgery + Radiotherapy

Correct Answer - B
Explanation
The management of cervical cancer depends on several factors, including the stage and extent of the disease,
the patient's age and overall health, and their personal preferences. In the case you presented, a 53-year-old
patient diagnosed with cervical cancer with a necrotizing lesion on the cervix that extends to the parametrium
would be classified as Stage IIIB cervical cancer.

The recommended treatment for Stage IIIB cervical cancer typically involves a combination of chemotherapy
and radiation therapy, known as chemoradiation. Chemoradiation is usually the preferred treatment option
over surgery for advanced cervical cancer because it has been shown to be effective in controlling the disease
and improving survival rates.

Q. 198 - A 25-year-old female who was sexually active with multiple sexual partners presents with swelling in
the wall of labia majora. What is the most likely diagnosis?

A. Bartholin abscess B. Cystic fibrosis


C. Gartner's cyst D. Trichomoniasis

Correct Answer - A
Explanation
135
Based on the symptoms mentioned, a Bartholin abscess is a likely diagnosis in this 25-year-old female who is
sexually active with multiple partners and has swelling in the wall of the labia majora.

The Bartholin glands are located on either side of the vaginal opening and produce a fluid that helps to lubricate
the vagina. When the ducts that drain the Bartholin glands become blocked, the fluid can accumulate and form
a cyst. If bacteria enter the cyst and cause an infection, an abscess can form.

The most common symptoms of a Bartholin abscess include swelling, pain, redness, and tenderness in the area
of the Bartholin gland.

Treatment for a Bartholin abscess typically involves draining the abscess and prescribing antibiotics to clear the
infection. In some cases, a small catheter may be placed to keep the abscess draining and prevent it from
recurring. In more severe cases, surgical removal of the Bartholin gland may be necessary.

Orthopedics

Q. 199 - Shown in image is an orthopedic procedure being performed. Identify the same?

136
A. Intraosseous cannulation for fluid therapy B. Intraosseous abscess drainage
C. Diagnostic biopsy D. Intraosseous cannulation for pain relief

Correct Answer - A
Explanation
Intraosseous infusion (IO) is the process of injecting medications, fluids, or blood products directly into the
marrow of a bone; this provides a non-collapsible entry point into the systemic venous system.

Q. 200 - The most likely diagnosis from the image given below is-

A. Paget's disease B. Osteogenesis imperfecta


C. Fibrous dysplasia D. Osteosarcoma

Correct Answer - B
Explanation
Osteogenesis imperfecta (OI) is an inherited (genetic) bone disorder that is present at birth. It is also known as
brittle bone disease. A child born with OI may have soft bones that break (fracture) easily, bones that are not
formed normally, and other problems. Signs and symptoms may range from mild to severe.

Q. 201 - Correctly matched statement regarding the splint shown in image-

137
A. Knuckle bender splint; UNP B. Dynamic cock up splint; RNP
C. Turn buckle splint; VIC D. Knuckle bender splint; MNP

Correct Answer - B
Explanation

FEATURES & BENEFITS


● It helps in avoiding flexion contracture in stroke patients.
● Used in Radial Nerve Palsy as a static splint For Spastic Hand/Rheumatic/Arthritic Disease.
● It holds the wrist and MP joints (Large Knuckles) of the fingers straight to reduce stress on muscles and
tendons, making it a great night splint for arthritis sufferers.
● Flexible padded stays protect the fingers to lessen the damaging effects of arthritis and certain neurological
impairments.
● The wrist and the metacarpal joints are supported in a neutral position.
● Suitable for skin contact, does not absorb perspiration.
● It does not support micro-organism growth.

Q. 202 - A patient C/O that he often gets troubled at night during sleep by tingling and numbness that
involves his lateral three digits. He further explains that symptoms are relieved as he lays his arms hanging
from the bed. Correct regarding the Condition

138
A. CTS: Froment sign B. CTS: Durkan test
C. Guyon canal syndrome: Froment sign D. Guyon canal syndrome: Durkan test

Correct Answer - B
Explanation
Carpel Tunnel syndrome
Durkan's test is a medical procedure to diagnose a patient with carpal tunnel syndrome.

139

You might also like